GRAMMAR-READING class 10

 

 

  1. READING

A1.1 Read the poem and answer the questions that follow:  

 

THE LEADER

 

  1. Patient and steady with all he must bear,

Ready to meet every challenge with care,

Easy in manner, yet solid as steel,

Strong in his faith, refreshingly real.

 

  1. Isn’t afraid to propose what is bold,

Doesn’t conform to the usual mould,

Eyes that have foresight, for hindsight won’t do,

Never backs down when he sees what is true,

Tells it all straight, and means it all too.

 

  1. Going forward and knowing he’s right,

even when doubted for why he would fight,

Over and over he makes his case clear,

reaching to touch the ones who won’t hear.

Growing in strength he won’t be unnerved,

 

  1. Ever assuring he’ll stand by his word.

Wanting the world to join his firm stand,

Bracing for war, but praying for peace,

Using his power so evil will cease,

So much a leader and worthy of trust,

  1. Here stands a man who will do what he must.

 

Q1.    The above poem refers to _________________________.

Q2.    ‘Doesn’t conform to the usual mould’ suggests the person being described is ________.

Q3.    The true qualities of a true leader are ____________ and ____________(any two)

Q4.    The leader would fight war bravely but __________.

Q5.    Using his power so evil will cease: Here cease means _______.

Q6.    Find the antonyms of the following words from the passage

  1. Insight [lines 5-7]
  2. Conflict [lines 15-17]

Q7.    Find the synonyms of the following words from the passage:

  1. Accurate [lines 8-10]

 

A1.2.   Read the passage and answer the questions that follow: 

 

Dharam Dev Pishorimal Anand (26 September 1923 – 3 December 2011), better known as Dev Anand, was an Indian film actor, writer, director and producer known for his work in Hindi cinema. Part of the Anand family, he co-founded Navketan Films in 1949 with his elder brother Chetan Anand.

 

The Government of India honoured him with the Padma Bhushan in 2001 and the Dadasaheb Phalke Award in 2002 for his contribution to Indian cinema. His career spanned more than 65 years with acting in 114 Hindi films of which 104 have him play the main solo lead hero and he did 2 English films. Dev Anand’s autobiography “Romancing with life” appears to be a very honest portrayal of the man called Dev Anand. This article is composed on the basis of revelations recorded in his life story. Being a very shy boy Dev’s father put him up in a girl’s school in Gurdaspur. It is obvious that Dev had a very captivating face.

 

As a child Dev was fond of playing with marbles on the street outside his house. He was an  excellent marksman from any distance. He was always sure of hitting every marble that he aimed for. Due to his marksmanship, he had won several marbles and stored those in a big jar, which was his proud possession. His father hated him for playing all day with marbles. Dev was afraid of his father. One day his father admonished him for playing with the marbles all the time. He said that this was not the way to attain stature in life. But he loved his mother very much.

 

While Dev was still in Gurdaspur, his mother developed Tuberculosis, a fatal disease during those days. The rare medicines necessary for her treatment were unavailable in Gurdaspur. Dev and friend Bhagoo used to go to Amritsar, more than thirty miles away from Gurdaspur, by bus to bring medicines for the treatment of his mother. Dev was fond of a special “Lassi” made from full fat milk, which used to have “Pedas” crushed into it. One sultry summer day Dev was sweating outside the Golden Temple in Amritsar. A Sikh gentleman was selling “Almond Sherbat”.  Dev put his hand forward to grab the tumbler of “Sherbat”. The Sikh “Sherbatwala” saw the unique blessings of sun on Dev’s forehead. He quickly said that someday you will be a big shot in life. Dev narrated this to his mother, who hugged him and told his father to give him the finest education and other facilities so that her son gets what he aspires for. His mother soon became too weak to walk even and was moved to a sanitarium, where she died.

 

Dev was enrolled in Government College Lahore for his graduation, which he did with honours in English. But soon he discovered that his father had fallen on bad days. Dev wanted to go to England for higher education, so that he could get an elite government job on return to India, but his father admitted that he could not afford this. His father gave him the option to do his master’s degree from Lahore Government College and then serve as a clerk in a bank, which Dev declined.

 

Q1.    Give a suitable heading for the above passage.

Q2.    The name of Dev Anand’s biography is _____________________.

Q3.    In his childhood he loved playing ______ and he stored them in a ______because they were his proud possession.

Q4.    He travelled to Amritsar with his friend Bhagoo, which was thirty miles away from his home in order to ____________________.

Q5.    The special lassi which Dev was particularly fond of was made of  ______________.

Q6.    Dev could not go to England to pursue his higher education because ___________.

Q7.    The Sikh sherbatwala, outside the Golden temple, told Dev that he would_________.

Q8.    From the passage, find the synonyms of the following word:

  1. a) story of your life (para 2)

 

 

A1.3.    Read the passage and answer the questions that follow :

 

WELCOME BACK YUVI ….!

 

Indian all-rounder and World Cup hero Yuvraj Singh will don national colours for the first time since battling cancer when a two-match Twenty20 series against New Zealand starts on Saturday. The 30-year-old left-hander underwent chemotherapy in the United States in March and April to treat a rare germ-cell tumour between his lungs which was diagnosed late last year.

 

Yuvraj, who was ‘Man of the Tournament’ in India’s World Cup triumph last year, has not played competitive cricket since two home Tests against the West Indies last November. But the selectors recalled him as soon as he was declared fit by doctors at the National Cricket Academy in Bangalore where he had begun light training in July.

 

In less than 36 hours from now, Yuvraj Singh will complete an incredible journey- that of having recovered from cancer and walking back on the field as an Indian cricketer. How many runs he scores is a different matter, it is his return to the field that makes him a winner.

 

On Saturday, Yuvraj will play his first international match after being diagnosed with cancer. And this journey was not an easy one. This was one test that took a lot out of him. “There was a lot of tension. There were negative thoughts in my mind. I used to cry a lot,” Yuvraj reminisces.  But all this while his teammates on the cricket pitch played the perfect mates off the field as well. “One day Anil Kumble came to meet me in Boston. He closed my laptop and said ‘stop watching cricket and focus on your health’,” Yuvraj said.

 

The left-hander did what he knows best – fought back! And soon the hero was back in India. It was a slow recovery- from stepping into the gym to stepping into the nets.

 

On Saturday, the journey will reach its most important phase. Yuvraj will be back in the India shirt, playing a T20 International against New Zealand. And he can’t wait for the match to begin. He landed in Vizag yesterday and tweeted: “Just landed in lovely Vizag!! Beautiful scenic view before landing! Hope it doesn’t rain tomorrow and day after!! Because I just can’t wait anymore.”  And he had wishes pouring from all corners. Bollywood superstar Shah Rukh Khan wished his friend good luck. He said, “I will repeat what Yuvraj said. It doesn’t matter if he scores one run or 20 runs or 200 runs. I wish he again hit six sixes. Whatever, he said, he has won and he actually has won. I would watch the match just because Yuvraj will be playing it.” Olympic silver-medalist MC Mary Kom also wished the southpaw, who won India U-19 World Cup in 2000, T20 World Cup in 2007 and ICC World Cup in 2011.

 

“I wish all the best in the future to Yuvraj,” Mary said. With the nation behind him, I expect the all-rounder to perform as good in his second innings as the first if not better.

 

Q1.    Yuvraj shed tears because

a.___________________    b.____________________

Q2.    Who came to meet him in Boston and what did he advise Yuvraj?

Q3.    Yuvraj will be playing a T20 International against __________in ______.

Q4.    What did Yuvraj hope for as soon as he landed in Vizag?

Q5.    Find the antonym of the word ‘loser’ from the passage.

 

A1.4.                                        NURTURING TALENT

 

Creative children usually possess strong creative needs; their interests are unexplainable and are naturally deeply hidden in them. These children are inquisitive ‘show interest in explaining things of fancy and test novel ideas that strike them. They do not accept ideas without questioning and verifying them. Creative children in most school in India feel neglected. Many children are unable to withstand pressure from parents and teachers, to be like other children in the school. Parents, in particular, want their children to do well in studies, secure good marks and grades. In these days of competition, they force them to get along with the schoolwork and prevent these children from using their creative abilities. We often hear from the parents of gifted children that they would be happy to see their children as higher achiever. Even the teachers in the schools admit that their aim is to reduce variations among the children in their classroom.

 

Creative children look to school and teachers for guidance and encouragement. Teachers should feel that creative children are of great values and they can become assets to the institutions. Creativity is the ability which is most valued in all societies. Constant encouragements given by the school helps these children in exhibiting their inborn abilities and skills/ The creative child’s hidden talent can be identified from an early age itself/ His choice of friends, hobbies, activities and dresses exhibits his/her inborn abilities. Creative and gifted children can master fundamental skills with minimum levels of training and they need help in understanding their strengths. These children believe that they are pursuing what they presume to be really worthwhile. If there is a teacher who can play the positive role of a facilitator, to kindle their creativity at an early age, wonderful results can be achieved. The school environment provides positive stimulus in exciting the creativity among these children.

 

The school counsellor (if any) may also help the parents to orient their attitude towards these children. No doubt the curious questioning of these children is very inconvenient to the parent. Nowadays many parents do intentionally prevent these children to learn on their own. One of the dominant personality traits among the creative children is independence. Independence in doing what they believe. These children possess the skills of improvisation and are always opened to new experiences. These children are not able to make something out of nothing. The act of creation involves a reshaping of a given material, either physically or mentally. A non-authoritarian, preferably pervasive, stimulating school environment is a positive input in  nurturing creativity. Teachers must set challenging tasks and encourage pupils towards working for unusual solution. Guiding children systematically to test new ideas is also very essential. Teachers should encourage the acquisition of new knowledge from diversified areas to develop constructive criticism. If the creative child is to maintain his/her creativity and continue to grow, he/she would need help from his parents and teachers for understanding and accepting his unique talents.

 

 

  1. In the following exercise, fill in the blanks with appropriate words or phrases.

(a)      The qualities of creative children are:

(i) Inquisitives

(ii) ________________

(iii) They test novel ideas

(iv) __________________

(b)      In these days of competition, creative children are forced to pursue their       studies to the detriment of ______________

(c)      Independence loving children possess the skills of _____________

(d)      Development of constructive criticism should be encouraged by the teacher in  the____________

(e)      __________by parents and teachers will lead to creativity and growth of their child.

 

  1. From the passage find words which mean the following:

(a)      A useful or valuable thing. (para 2)

(b)      Something that promotes activity, interest or enthusiasm. (para 2)

(c)      Produce or make something from whatever is available.(para 3)

 

 

A1.5.                                        HOW MANY MORE?

 

All is quiet in this vast Himalayan jungle except for the occasional call of the hornbill. As the group of forest officials treads gingerly ahead in search of poachers, a stench begins to rise  from the bowels of the jungle. The winding track dips into a leafy creek. No humans here, just the putrefying half-eaten body of a bull at Paterpani in the Core Zone of the Corbett National Park on 8th February. Fresh pug marks suggest that tigers have been approaching the dead bull, Bhanda, regularly. Above them circles a flock of hungry vultures ready to feast on the remains after the tigers depart. A series of daring strikes in the past three months resulted in five elephants following prey to a powerful poaching mafia which has spread its tentacles in the supposedly well-guarded wildlife sanctuary. Trailing the poachers is a tough task as Brijendra  Singh, the park’s honorary wildlife warden who has spent the past twenty years preserving it, will testify. Singh is the driving force behind the 150-odd forest guards who undertake daily missions into the heart of the jungles. He wants the poachers-probably numbering only five but  ‘highly skilled at jungle craft-stopped an any cost.’ In a desperate bid to isolate the poachers, officials closed the parks for a day and even used helicopters to search for poachers, but to no avail. Now the CBI too has joined the hunt.

 

The urgency to pin down the hunters is mounting as the poaching mafia is increasing striking at will all across the country. Between July 1998 and October 1999, about a dozen tuskers were poached in the forest of Coochbehar in West Bengal. The modus operandi was the same as that Corbett. The poachers are interested in the ivory which fetches more than Rs 50,000/- per kg in the international market, the ban on ivory trade having been lifted. A tusker on an average yields 15 to 20 kg of ivory. In 2000 alone, an estimated 100 elephants fell to the avaricious poachers in the various sanctuaries signaling an escalation of a trend that had been subdued for much of the 1990s. For the past three years, elephant mortality is touching the soaring levels the notorious Veerappan had taken it to in the southern ranges in the 1980s.

 

With Veerappan on the run, his role has been usurped by dozens of group who usually operate independently and chalk out their own turf. But the Corbett killings have shown that there may be alarger group operating on a much wider scale. Singh has dubbed it the ‘Chisel Gang’ for their unique method of hunting. It is simple, but deadly. The poachers put bait for the pachyderms armed with muzzle loaders. When they spot a tusker, a 6cm long chisel-like iron dart soaked in lethal pesticides is fired from those proximity into the animal’s under belly.

 

 

In the following exercise, fill in the blanks with suitable words or phrases.

 

(a)      The animals circling the remains of the dead bull Bhanda are ____________

(b)      ‘Highly skilled at jungle craft’ means _______________

(c)      The poachers hunt the elephants for _______________ per kg in the international market.

(d)      _______________seems to have taken to elephant poaching in the 1980s.

(e)      ‘Chalk out their own turf’ means ________________

(f)       The Chisel Gang fires a 6cm long, chisel-like iron dart soaked in lethal pesticides____________

(g)      Brijendra Singh calls the gang __________ for their unique method of hunting.

 

 

  1. From the passage find words which mean the following:                                          

(a)      Greedy (para 2)

(b)      Deadly (para 3)

 

 

A1.6.                                        The Photograph

-Ruskin Bond

 

I was ten years old. My Grandmother sat on the string bed under the mango tree. It was late summer and there were sunflowers in the garden and a warm wind in the trees. My grandmother was knitting a woollen scarf for the winter months. She was very old, dressed in a plain white sari; her eyes were not very strong now, but her fingers moved quickly with the needles, and the needles kept clicking all afternoon. Grandmother had white hair, but there were very few wrinkles on her skin.

 

I was rummaging in a box of old books and family heirlooms that had just been brought out of the attic by my mother. Nothing in the box interested me very much except for a book with colourful pictures of birds and butterflies. I was going through the book, looking at the pictures, when I found a small photograph between the pages. It was a faded picture, a little yellow and foggy. It was the picture of a girl standing against a wall and behind the wall there was nothing but sky : but from the other side a pair of hands reached up, as though someone was going to climb the wall.

 

I ran out into the garden, ‘Granny’, I shouted, Look at this picture! I found it in the box of old things. Whose picture is it?’  She took the photograph from my hand, and we both stared at it for a very long time.

‘Whose picture is it?’ I asked.

 

‘A little girl’s, of course,’ said Grandmother. ‘Can’t you tell?’

 

“Yes, but did you know the girl?”

 

‘Yes, I knew her,’ said Granny, ‘but she was a very naughty girl and I shouldn’t tell you about her. But I’ll tell you about the photograph It was taken in your grandfather’s house about sixty years ago. And that’s the garden wall and over the wall there was a road going to town.’

 

‘Who was the girl?’ I said/ ‘You must tell me who she was.’

 

‘No, that wouldn’t do,’ said Grandmother. ‘I won’t tell you.’

 

I knew the girl in the photo was really Grandmother, but I pretended I didn’t know. I knew because grandmother still smiled in the same way, even though she didn’t have as many teeth.

 

‘Come on, Granny,’ I said, ‘tell me, tell me.’

 

But grandmother shook her head and carried on with the knitting. And I held the photograph in my hand looking from it to my grandmother and back again, trying to find points in common between the old lady and the little pig-tailed girl. A lemon-coloured butterfly settled on the end of Grandmother’s knitting needle and stayed there while the needles clicked away. I made a grab at the butterfly and it flew off in a dipping flight and settled on a sunflower.

 

 

Q1.    The grandmother was busy____________.

Q2.    ____________  were blooming in the garden.

Q3.    The boy found _______ in the box of  old things.

Q4.    The photograph was taken _______ ago at the boy’s ________.

Q5.    Apparently, the girl was ________ in the photograph.

Q6.    The boy recognized the young girl in the photograph by____________________.

Q7.    Find words in the passage which mean the following:

  1. a) Searching [para 2]
  2. b) Falling [last para]

 

 

A1.7.                                                 TRUE GREATNESS

 

  1. Paderewsky was a rich man gifted with an ear for music. With his aptitude and with the help of tutors, in time, he became a great musician. He was a wizard with the violin. People thronged to hear his recitals, critics acknowledged him as a master violinist. He accepted the laurels heaped on him because he knew and realized the power of his music. Alas, success had made him proud. He felt that he was the only musician who could translate any emotion or render any tune on his violin. One day, while out on a morning walk in the woods he sat on a stone to admire nature. He felt that nature was all set to teach him a new tune of divine joy. The wind caused a gentle rustle of leaves and it seemed like the opening bars of a symphony. A few twigs fell, striking a strong note. There was a pause – a hush. Then a tiny sparrow started trilling a sweet song of gratitude to its maker, lifting its heart to heaven. The music of the swaying flowers and the enchanting song of the unassuming singer lulled and soothed the musician. It stirred the innermost recesses of his heart. He knew that he must render the same piece of music on his violin. The song ended and the bird flew away.

 

  1. The musician jumped up, elated. He rushed home excited. What a great tune nature had presented to him. He would render it on his violin for his performance that very evening. Evening came and the music hall was packed. Paderewsky went on stage and bowed to the audience. The accompanist played the opening bars. People waited with bated breath to catch the first notes of the great master. The artist smiled loftily and drew his bow lightly across the strings. But something unexpected had happened. He had forgotten the song of the bird completely. The tune he had heard only that morning had gone out his mind. Irritated, he tried again but only succeeded in making a few screeching noises. The audience grew restless. Some even laughed. Paderewsky felt humiliated and angered. He flung the violin, it smashed against the wall and broke. Paderewsky looked up dejected. The hall was empty. He had paid a heavy price for his vanity. Tears flowing he realized that even the humble sparrow was greater than him. The greatness of a person is not measured by the talents he or she has. It is not measured by the position one holds. It is never measured by the popularity or clout one has. It is measured by one’s humility and good deed.

 

 

  1. In the following exercise, fill in the blanks with suitable words or phrases.

 

(a)      The two examples of Paderewsky’s success as a musician were

(i)       People thronged to hear his recitals

(ii)      __________________

(b)      Paderewsky did not_________because he was conscious of the power of his music.

(c)      As a result of his pride he assumed that he was the best musician who could ___________________.

(d)      One day, while out on a morning walk he sat on a stone to admire nature which  was all set to teach him a_________________

(e)      The two things that lulled and soothed the musician were

(i)       ______________

(ii)      The enchanting song of the humble sparrow.

(f)       The song of the bird influenced the musician so much that he felt____.

(g)      Something unexpected had happened. Paderewsky had forgotten________

(h)      The greatness of a person is measured not by his talents or position or popularity, but_____________

 

A1.8.                                                 Rest in peace

 

  1. Who climbed Mount Everest first? Edmund Hillary and Tensing Norgay set foot on the highest peak in the world in 1953, the year of the coronation of Queen Elizabeth II. But there is another school of thought, a more passionate one that gives the credit for the mountaineering feat to a team of two young daring Britons, George Leigh Mallory and Andrew Irving. They were supposed to have reached the summit of the Everest on 8thJune 1924, more than three decades before Hillary and Norgay reached the base of the mountain. There is only one problem with this theory: neither Mallory nor Irving lived to tell the tale of their success on the treacherous mountain. They disappeared somewhere close to the summit, some say only 180 metres from the bald patch of snow and ice, an uninspiring feature closer to the heavens than any other point in the world, which has attracted numerous intrepid souls across the world for decades. Therein hangs one of the most enduring mysteries of the mountaineering world by all accounts. Mallory and Irving were the unlikeliest of men to become partners in an adventure like an Everest expedition. Mallory was an experienced mountaineer, having honed his skills in the Alps and other European mountains Irving was younger to him and inexperienced. When they met, Everest was talked about in hushed tones in pubs where mountaineers often gathered to wind down and exchange stories. It was in a distant world. Nothing much was known about it except that the locals called it Sagarmatha and worshipped it as the holy seat of the Mother Goddess. What intrigued the mountaineers most was the British Surveyor General, George Everest, calling it, in the 19th  century, the highest mountain in the world. Until then, Europeans had never imagined there could be mountains higher than the Alps.

 

  1. The legend of the Everest was born thus. It fired the imagination of mountaineers of the world over. Mallory and Irving were just two of them who set forth for the distant mountain. The news of their success in opening the route across some of the most treacherous portions of the Everest route excited the mountaineering world to no end. It was the first time anyone had climbed to such heights. Even before dawn broke on 8th June 1924, Mallory and Irving began their journey to the summit. They were last seen then. Many said they fell to God’s wrath having defiled the holy seat of the Mother Goddess. Some said it was Irving’s inexperience which caused the tragedy. There were talks of the Yeti killing them. And yet there were others who argued that the duo never went anywhere near the summit and that it was all part of an imperialist conspiracy.

Two years ago, American guide Eric Simonson found Mallory’s frozen body some 180 metres from the summit and set at rest all such speculations. This year, he is planning another expedition to recover the remains of Irving. But he has run into stiff oppositions from Irving’s family which wants their hero to remain in the shadow of summit which, even in the day of space exploration, remains an enduring symbol of adventure and discovery.

 

  1. In the following exercise, fill in the blanks with suitable words or phrases.

(a)      Young and daring Britons, George Leigh Mallory and Andrew Irving are                             credited __________ more than three decades before Hillary and Norgay.

(b)      Mallory and Irving were the unlikeliest of men to become partners in                       adventures like an expedition to the Everest because __________

(c)      The Everest was called ____________of the Mother Goddess.

(d)      General George Everest called it the_______in the world, higher than _____

(e)     There are many curious about the disappearance of Mallory and Irving. Some of the these theories are:

(i)       _______________

(ii)      _______________

(ii)      Some say the yeti killed them.

(iv)     Some say it was a part of an imperialist conspiracy.

(f)       American guide Eric Simonson found Mallory’s frozen body  some _______.

 

  1. From the passage find a word which means the same as:

fearless/adventurous(para1)

 

 

2A.   Read the passage given below.

 

When I was at college I used to spend my summer vacations in Dehra, at my grandmother’s place. I would leave the plains early in May and return in July. Deoli was a small station about thirty miles from Dehra: it marked the beginning of the heavy jungles of the Indian Terai.

 

The train would reach Deoli at about five in the morning, when the station would be dimly lit with electric bulbs and oil-lamps, and the jungle across the railway tracks would just be visible in the faint light of dawn. Deoli had only one platform, an office for the station master and some stray dogs; not much else, because the train stopped there  for only ten minutes before rushing on into the forests.

 

Why it stopped at Deoli, I don’t know. Nothing ever happened there. Nobody got off the train and nobody got in. There were never any coolies in the platform. But the train would  halt there a full ten minutes, and then a bell would sound, the guard would blow his whistle, and presently Deoli would be left behind and forgotten.

 

I used to wonder what happened in Deoli, behind the station walls. I always felt sorry for that lonely little platform, and for the place that nobody wanted to visit. I decided that one day I would get off the train at Deoli, and spend the day there, just to please the town.

 

I was eighteen, visiting my grandmother, and the night train stopped at Deoli. A girl came down the platform, selling baskets.

 

It was a cold morning and the girl had a shawl thrown across her shoulder. Her feet were bare and her clothes were old but she was a young girl, walking gracefully and with dignity.

 

When she came to my window, she stopped. She saw that I was looking at her intently, but at first she pretended not to notice. She had a pale skin, set off by shiny black hair, and dark, troubled eyes. And then those eyes, searching and eloquent, met mine.

 

She stood by my window for sometime and neither of us said anything. But when she moved on, I found myself leaving my seat and going to the carriage door. She noticed me at the door, and stood waiting on the platform, looking the other way. I walked across to the tea stall. A kettle was boiling over a small fire, but the owner of the stall was busy serving tea somewhere on the train. The girl followed me behind the stall.

 

‘Do you want to buy a basket?’ she asked. ‘They are very strong, made of the finest cane’.

 

‘No,’ I said, ‘I don’t want a basket’.

 

We stood looking at each other for what seemed a very long time and then she said, Are you sure you don’t want a basket?’

 

‘All right, give me one,’ I said, and I took the one on top and gave her a rupee, hardly daring  to touch her fingers.

 

Q2A. Read the questions given below and choose the option that you think is the most appropriate.

 

  1. i) At this stage of his life the writer was
  2. a) a young man                        b) an old man
  3. c) a child                                 d) middle aged

 

  1. ii) His summer vacation was special as he spent it at his
  2. a) Hostel                 b) home
  3. c) grandmother’s place             d) friend’s home

 

iii)      The writer left his seat to

  1. a) Close the window b)  see the girl
  2. c) meet the guard                  d) get down

 

  1. iv) The girl carried with her some
  2. a) Flowers                      b) fruits
  3. c) baskets                       d) cane

 

  1. v) The writer was attracted to the girl because of her
  2. a) bare feet                              b) baskets
  3. c) grace and dignity                 d) dark troubled eyes

 

  1. vi) The word “visible” means
  2. a) seen                  b)found

c)noticed                                   d) wondered

 

vii)     The Writer and the girl stood near a

  1. a) Bookstall                b)Water tap

c)Tea stall                                 d)Window

 

 

2B.    Read the following passage and answer the questions –                         

 

This is the story of Phineas Snodgrass, inventor. He built a time machineand in it he went back some two thousand years to about the time of the birth of Christ. He made himself known to Emperor Augustus, his lady Livia and other powerful and rich Romans of the day and, quickly making friends, secured their cooperation in bringing about a rapid transformation of yearlong living habits. (He stole the idea from a science fiction novel by L. Sprague De Camp called  “Lest Darkness Falls’’.

 

His time machine wasn’t very big, but his heart was. So Snodgrass selected his cargo with the plan of providing the maximum immediate help for the world’s people. The principal features of ancient Rome were dirt and disease, pain and death. Snodgrass decided to make the Roman world healthy and to keep its people alive through twentieth century medicine. Everything else could take care of itself, once the human race was free of its terrible plagues and early deaths.

 

Snodgrass introduced   penicillin and Aureomycin   and painless   dentistry.  He ground lenses for spectacles and explained the surgical techniques for removing cataracts. He taught anaesthesia and the germ theory of disease. And showed   how to purify drinking water .He demanded, and   got, covers for the open Roman sewers, and he pioneered the practice of the balanced diet.

 

Q2 B.           Read the questions given below and choose the option which you think is the most appropriate:        

 

  1. i) Phineas Snodgrass built a
  2. a) Spaceship                           b) Aircraft
  3. c) Time machine                      d) Time selector

 

  1. ii) He quickly made friends in order to
  2. a) secure himself                     b)secure cooperation

c)write a novel                          d)meet Emperor Augustus

 

iii)      Snodgrass decided to make Romans

  1. a) Educated b)  Learn warfare

c)Healthy and alive                   d) Dirty and diseased

 

  1. iv) Plague means
  2. a) an infectious and fatal disease b)Death
  3. c) Unhygienic conditions    d )Painful death

 

  1. v) Snodgrass was a
  2. a) Painter                                b) Scientist
  3. c) Bone collector d) Builder

 

  1. vi) The Principal features of Rome was

a)Health and  hygiene               b)Dirt and disease

  1. c) Cooperation                         d) Rapid transformation

 

vii)     He wanted to use the medicine of

a)76 B.C.                                  b) 76 A.D

c)20th   Century                        d)  16th  Century.

 

2C.    Read the following passage and answer the questions     

 

If you meet a member of that select club, “the Twelve True Fishermen”, entering the Vermon  hotel for the annual club dinner, you will observe, as he takes off his overcoat, that his evening coat is green and not black. If you ask him why, he would answer that he does it to avoid being mistaken for a waiter. You will then retire crushed. But you will leave behind a mystery as yet unsolved and a tale worth telling.

 

If you were to meet a mild, hardworking little priest, named Father Brown, and were to ask him what he thought was the most singular luck of his life, he would probably reply that upon the whole his best stroke was at the Vermon Hotel, where he had averted a crime and, perhaps, saved a soul, merely by listening to a few footsteps in a passage.

 

The Vermon Hotel, at which The Twelve True Fishermen held their annual dinners, stood, as if by accident, in the corner of a square in Belgravia. It was a small hotel; and a very inconvenient one. But its very inconveniences were considered as walls protecting a particular class. One inconvenience, in particular, was held to be of vital importance: the fact that practically only twenty-four people could dine in the place at once. The only big dinner table was the celebrated terrace table, which stood open to the air on a sort of verandah overlooking one of the most exquisite old gardens in London. Thus it happened that even the twenty four seats at this table could only be enjoyed in warm weather; and this made the enjoyment more difficult yet more desired. The existing owner of the hotel was a Jew named Lever; and he combined with his limitation in the scope of his enterprise the most careful polish in his performance.

 

The wines and cooking were really as good as any in Europe. , and the demeanour of the attendants exactly mirrored the fixed mood of the English upper class. The proprietor knew all his waiters like the fingers on his hand; there were only fifteen of them, all told. It was much easier to become Member of Parliament than to become a waiter in that hotel. Each waiter was trained in terrible silence and smoothness, as if he were a gentleman’s servant/ And, indeed, there was generally at least one waiter to every gentleman who dined.

 

2C.     Read the questions given below and choose the option which you think is the most appropriate:

 

  1. i) The “Select club” stands for
  2. a) The Vermon hotel b) The twelve true fishermen
  3. c) Terrace table                    d) English upper class

 

  1. ii) Their evening coat is green in order to
  2. a) be environment friendly   b) avoid being mistaken for a waiter
  3. c) be identified easily as waiters         d) avoid inclement weather

 

iii)      The proprietor knew all the waiters as they

  1. a) Were his relatives b)were few in number

c)lived with him                        d)met him often

 

  1. iv) “Precisely” means
  2. a) Exactly                                b) Suddenly
  3. c) Hopefully                             d) Vainly

 

  1. v) The hotel was famous for its
  2. a) Select club                          b )Warm weather
  3. c) Beauty                                 d) Hospitality

 

  1. vi) The term that the Writer uses for the waiters is
  2. a) Member of Parliament       b) Gentlemen
  3. c) Gentleman’s servant             d) Select  Club

 

vii.     The name of the Hotel owner is

  1. a) Lever                    b)Jew

c)Belgravia                               d) Jim

2D.    Read the following passage and answer the questions :–              

 

The Hindu Code Law was duly passed but it did not provide beleaguered women respite from the violence that gripped their lives. Even now, thanks to a disgraceful social practice, many women lead lives of anguish, often finding respite only in death. The name of this practice is dowry and many women, irrespective of whether they are poor or rich, are its hunted victims. The law prohibiting such practices had been passed much earlier in the Lok Sabha. But one must remember that the Congress could only get the law passed by applying the party whip on its own members. In other words, even within the Congress there were differences of opinion. In the opposition too  the orthodox fought against the bill.

 

When the law was being hotly debated, we too arranged many meetings and gatherings among women and collected many signatures in its support. We found that many poor housewives in towns and villages and many peasant women did not speak up against the bill. This was primarily out of fear if women couldn’t get married without paying a dowry, then what was the point of supporting this law? During our sessions we heard how much land peasant families and/ or lower and upper caste families had to give up and how many had become bankrupt in trying to get their daughters married. Still they didn’t have the courage to state, “No, we shall not pay dowry anymore”

 

But if the promised  amount  could   still  not be paid within the given period ,in certain cases ,the new bride fell victim  to the violent attacks of her husband and parents in law. Such incidents did not take place only in poor families but also   amongst  the   rich.  The   groom’s family often didn’t set limits to its cruelty – the bride could be killed or driven to suicide. His family dreamt of a new marriage for the groom with new dowry. Such incidents are often published in newspapers today. Even the rich have joined the anti – dowry  processions.

 

2D.    Read the questions given below and choose the option that you think is the most appropriate –

 

  1. i) The Hindu code law was made to give respite to women from
  2. a) Violence                               b)Disgrace

c)Whipping                               d) Differences

 

  1. ii) In the passage ‘disgraceful social practice’ refers to

a )Poverty                                 b)Dowry

c)Violence                                 d) Untouchability

 

iii)      Orthodox means

  1. a) Extremely conservative b) Lawful
  2. c) Doubtful                              d) Opinionated

 

  1. iv) The Law had already been passed by
  2. a) Congress                              b) Lok Sabha
  3. c) Opposition                           d) Women
  4. v) Anguish means
  5. a) Pain and death                     b) Disgraceful
  6. c) Pain and suffering                d) Fight

 

  1. vi) The women did not speak against the bill due to

a )Shame                                  b) fear

  1. c) Pride                           d) Ignorance

 

vii)     The word Bankrupt means

  1. a) Without any money        b)cheating the bank

c)lending money                        d) saving money

 

 

Passage 1 (Solved)

 

TV AND OBESITY

 

US nutrition experts have come to the conclusion that watching too much TV was one of the main reasons why children in USA were overweight.

 

Wilhelm Dietz, a nutrition scientist at the National Centre for the Prevention of chronic diseases in Atlanta, Georgia, said that tests carried out at several schools in Massachusetts and at a clinic in Pittsburgh, Pennsylvania had shown clearly that children soon shed their excess pounds when they stopped spending so much time in front of the TV set.

 

The tests involved studying the eating and TV-watching habits of 1,295 schoolchildren in the sixth and seventh grades.  It was found that overweight children who reduced the time they spent watching TV by 20 hours per week, lost up to 20 per cent of their body weight in four months, and were able to maintain their new slimmer outline.

 

By comparison, another group of children who simply did more physical exercise during the same period lost only 13 per cent of their weight and after a degree of initial success, quickly put it back on again.

 

Presenting the results of the studies in New York, Wilhelm Dietz said that parents tended to underestimate the amount of time their children spent motionless in front of television sets, in many cases, they did not even know that their children were sitting chained to a TV programme at kindergarten or a friend’s house, for example. Pediatricians recommend that children should not watch TV for more than two hours per day.

 

“The more TV they watch, the more they tend to eat the things that are advertised on TV,” said Dietz. Unfortunately, it usually meant high-fat snacks, potato crisps or chocolate.

 

“The more TV children watch, the more lethargic they are,” said Dietz. He recommends that parents implement a kind of bonus strategy to encourage their children to adopt better habits, such as allowing them half an hour of TV for every hour they spend playing outdoors. And, said Dietz, on no account should they have a television set in their bedrooms.

 

  1. Excess TV viewing by children results in

(a) Being overweight

(b) Intelligence

(c) Disease

(d) Success

 

  1. Effective weight loss can be achieved by

(a) Eating less

(b) Sleeping more

(c) Physical exercise

(d) Cut in TV viewing time

 

  1. According to paediatricians children should watch TV for

(a) More than two hours per day

(b) More than three hours per day

(c) Less than one hour per day

(d) Less than two hours per day

 

  1. Overweight children lost 20 per cent of their body weight in four months when they reduced the time they spent watching TV

(a) by 5 hours per week

(b) by 10 hours per week

(c) by 15 hours per week

(d) by 20 hours per week

 

  1. The National Centre for the presentation of chronic Diseases in Atlanta studied the eating and TV-watching habits of

(a) School children in the fourth and fifth grades

(b) School children in the sixth and seventh grades

(c) School children in the eighth and ninth grades

(d) School children in the ninth and tenth grades

 

  1. The more TV the children watch

(a) The more they learn

(b) The less they sleep

(c) The more they eat

(d) The less they think

 

  1. The word lethargic means
  2. a) energetic

(b) knowledgeable

(c) lazy

(d) smart

 

Passage 2 (Solved)

 

RAIN ON THE ROOF

 

  1. When the humid shadows hover

Over all the starry spheres

And the melancholy darkness

Gently weeps in rainy tears,

 

  1. What a bliss to press the pillow

Of a cottage-chamber bed

And lie listening to the patter

Of the soft rain overhead!

Every tinkle on the shingles

 

  1. Has an echo in the heart;

And a thousand dreamy fancies

Into busy being start,

And a thousand recollections

Weave their air-threads into woof,

 

  1. As I listen to the patter

Of the rain upon the roof.

Now in memory comes my mother,

As she used in years agone,

To regard the darling dreamers

 

  1. Ere she left them till the dawn:

O! I feel her fond look on me

As I list to this refrain

Which is played upon the shingles

By the patter of the rain.

– Coates Kinney

 

  1. The dark wet shadows of clouds make the darkness look

(a) starry

(b) lovely

(c) sad

(d) fearful

 

  1. When it rains

(a)      The poet wants to play

(b)      The poet wants to sing

(c)      The poet wants to listen to the patter

(d)      The poet wants to sleep

 

  1. Every sound of rain on the shingles

(a)      Creates happiness in the heart

(b)      Creates sadness in the heart

(c)      Creates imagination in the heart

(d)      Creates an echo in his heart

 

  1. When the poet listens to the rain

(a)      He remembers his children

(b)      He remembers his mother

(c)      He remembers his dreams

(d)      He remembers his friends

 

  1. The word ‘ere’ in the stanza 3 means

(a ) dear

(b) foolish

(c) loving

(d) before

 

  1. As the poet listens to the refrain, he feels

(a) Pleasure

(b) the rain falling on him

(c) His mother’s presence

(d) he is dreaming

 

  1. The word ‘bliss’ in stanza 1 means

(a) blessing

(b) sadness

(c) great joy

(d) happiness

 

 

Passage 3(Unsolved – for Practice)

 

WATER: LIFE’S MAIN SOURCE

 

Read the following passage carefully and mark the option you consider most appropriate.

Man does not live by food alone. Water is vital to human health and fitness. Although it is not a nutrient per se as are carbohydrates, fats, proteins, vitamins and minerals. It, in fact, is a key nutrient in as much as no life is possible without it. Whereas we can do for weeks without food, we cannot live without water longer than a couple of days.

 

Water approximates 60 per cent of the body weight of human adults. The total amount of water in a man weighing 70 kilograms is approximately a little over 40 litres. It is an excellent solvent – more substances are soluble in water than in any other liquid known so far. This makes it an ideal constituent of the body fluids which sustain life supporting chemical reactions. It dissolves varied products of digestion and transports them to the rest of the body. Likewise, it dissolves diverse metabolic wastes and helps drain them out of the body. Besides, it performs a variety of functions-some well known and well understood while others not so well appreciated yet vital. The no less important role of water is to distribute/dissipate the body heat efficiently, thereby regulating the body’s temperature. Water accomplishes this role ideally because it has thermal conductivity ensuring rapid heat from one part to the other.

 

Above all, water has a high-specific heat, implying that it takes a lot of heat to raise the temperature of water and likewise much heat must be lost to lower its temperature. Drinking a lot of water is an inexpensive way to stay healthy. Even excess of water is harmless. Water therapy-drinking a litre or so the first thing in the morning is kidney-friendly. The water regulation in the body is affected by hypothalamus in two ways i.e.,(i) by creating the sensation of thirst which makes us drink water and (ii) by controlling the excretion of water and urine. If water regulation fails, medical emergency ensues.

 

  1. Man cannot live for more than a couple of days

(a) Without food  (b) without water  (c) without oxygen  (d) without fruits

  1. Water is an excellent solvent because

(i)       It regulates excretion of urine

(ii)      It dissolves metabolic wastes

(iii)     It drains wastes out of body

(iv)     More substances are soluble in it than in any other liquid.

  1. The high thermal conductivity of water helps to

(i)       Dissolve food

(ii)      Dissolve metabolic wastes

(iii)     Regulate body temperature

(iv)     Sustain life supporting chemical reactions

  1. The total amount of water in a man weighing 70 kg is

(i)       Approximately 70 litres

(ii)      Approximately 60 litres

(iii)     Approximately 40 litres

(iv)     Approximately 50 litres

  1. High specific-heat of water means

(i)       It has high thermal conductivity

(ii)      It takes less heat to raise its temperature

(iii)     It takes more heat to raise its temperature

(iv)     It distributes the body heat efficiently

  1. Drinking a litre of water in the morning is called

(i)       Hypothalamus

(ii)      Water regulation

(iii)     kidney therapy

(iv)     water therapy

  1. The word ‘ ideal’ means

(i)       lazy

(ii)      most suitable

(iii)     valuable

(iv)     good

 

 

Passage 4 (Unsolved – for practice)

 

THE DREADED DRUGS

“DRUGS” the most dreaded things of today,

Which are gaining notoriety day by day

Taking drugs is injurious,

Because Man’s life is very precious.

Learn to say ‘NO’ to drugs,

Because they are more dangerous than bed bugs.

Life is precious to all,

So knowingly don’t cut it small.

It is like a rat’s trap,

Don’t indulge in the mishap

Drugs are life-takers.

It is the greatest enemy of mankind,

Which spoils the brain and mind.

Don’t try to have its taste,

Otherwise your life will be waste

So why depend on drugs alone?

Which makes a family mourn.

Don’t allow drugs to make your life dull,

As it brings your great failure

Don’t take it under peer pressure,

As it spoils your future.

Prevent yourself from preventing the blunder,

Which will make you only wander.

Then why let drugs spoil your precious life,

When you can willingly throw away your cigarette pipe.

 

On the basis of your reading of the poem choose the most appropriate answer from the options given below:

 

  1. Mark the wrong statement

(a) The use of drugs is increasing day by day

(b) Taking drugs is beneficial

(c) We must say ‘No’ to drugs

(d) Drugs are dangerous

 

  1. According to the poet, drugs are more dangerous than

(a) wild animals                                  (b) alcohol

(c) bed bugs                                        (d) enemies

 

  1. ‘No’ to drugs means

(a) yes to life                                       (b) no to life

(c) more to life                                    (d) right to life

 

  1. A word in the poem similar in meaning to ‘ evil image ‘ is

(a) injurious                                       (b) dangerous

(c) bed bugs                                        (d) notoriety

 

 

  1. Drugs make your life

(a) bright                                            (b) sad

(c) dull                                               (d) happy

 

  1. According to the poet , the greatest enemy of mankind is

(a) bed bugs                                        (b) alien

(c) pollution                                        (d) drugs

 

  1. One is tempted to take drugs because of

(a) curiosity                                        (b) money

(c) parents                                          (d) peer pressure

 

 

Passage 5 (Unsolved – for Practice)

 

Read the following passage carefully and mark the option you consider the most appropriate.

 

Language is verbal presentation. Phonetic code of delivery makes rapid growth of culture, race, origin with modern age of technology causing reliable national growth. A new database for spoken English is being created at the Oxford University Phonetics Laboratory.

 

Professor John Coleman and his team are one of four teams to win the ‘Digging into Data’ competition set up to encourage imaginative, forward-thinking research using large-scale computing in Humanities.

 

The resulting database will contain a year’s worth of spoken English and the project mining a year of speech will create the world’s largest searchable database of spoken English Sound recordings.

 

It will be a useful resource for anyone interested in spoken English not just phoneticians and linguists, but also many other kinds of people such as teachers of English language, social historians, and interested members of the public.

 

Professor Coleman said, “In a world where there’s more multimedia than text, audio searching is becoming a vital technology, even Google is moving into it now. We will provide the data so that it is searchable, but we can’t even begin to imagine the full range of questions about language that people will want to use it for”

 

  1. A database enabling people to search for English sound recordings is being created by

(i)       Professor John Coleman

(ii)      Oxford University Phonetics Laboratory

(iii)     Google

(iv)     Phoneticians

  1. Digging into data aims at

(i)       encouraging research

(ii)      collecting data

(iii)     making English easy

(iv)     promoting English as an international language

 

  1. The new database for spoken English will benefit

(i)       students and teachers

(ii)      the phoneticians

(iii)     linguists

(iv)     all of the above

 

  1. Reliable national growth is the result of

(i)       Spoken English

(ii)      computers

(iii)     the modern age of technology

(iv)     the work done by historians

 

  1. ‘Digging into Data’ competition has been won by

(i)       one team

(ii)      two teams

(iii)     Professor Coleman

(iv)     four teams

 

  1. The study of sounds of a language is called

(i)       Grammar

(ii)      Literature

(iii)     Etymology

(iv)     Phonetics

 

 

ANSWERS TO COMPREHENSION PASSAGES

 

Passage 1:TV and Obesity

 

  1. (a) overweight
  2. (d) cut in TV viewing time
  3. (d) less than two hours per day
  4. (d) 20 hours per week
  5. (b) school children in the sixth and seventh grade
  6. (c) the more they eat
  7. (c) lazy

 

Passage 2 : Poem – Rain on the Roof

  1. (c) melancholy
  2. (c) the poet wants to listen to the platter
  3. (d) creates an echo in his heart
  4. (b) he remembers his mother
  5. (d) before
  6. (c) his mother’s look on him
  7. (c) great joy

 

 

 

 

 

NOTICE

Value Point for Contents

Events

  • Who is the notice for
  • Date and time, venue
  • Additional information
  • (like chief guest, topic, ticket)

Meeting

  • Who is the notice for
  • Agenda

Lost and Found

  • Who is the notice for
  • Date, time, venue where object was lost/found
  • Contents (if any)

Tours and Camps

  • Contents (if any)
  • Who is the notice for
  • Duration
  • Accommodation
  • Whom to contact
  • purpose
  • any deadline (if required)
  • date, time, venue
  • destination
  • mode of travel
  • charges and last date for
  • registration, permission letter

Making an Appeal

  • who is the notice for
  • whom to contact,
  • where and when
  • what is urgently required,
  • how to send/give the requirement for whom and why

 

Sample Notice

Q.1

You are Damini Ratnam, President – Debating Society. A.P.S.Dhaula Kuan, New Delhi. Your society is going to organize an Inter-School English debate competition. Write a notice to announce the event and to invite students of the school (classes IX to XII) to form the audience.

 

Ans.

ARMY PUBLIC SCHOOL

Dhaula Kuan, New Delhi

NOTICE

20 November 2014

INTER SCHOOL ENGLISH DEBATE

Our society is organizing an Inter-school English debate competition on the
24th. Fifteen prestigious schools of the Delhi are sending their teams to contest for the coveted running trophy.

Venue: Raiina Auditorium

Timings : 9 a.m. to 1p.m.

Students of senior classes (IX to XII) are cordially invited to attend.

Damini Ratnam

(DAMINI RATNAM)

President Debating Society.

 

 

Q.2.

You are Romit / Romita, CCA Captain of Students’ council of Sacred Heart School, Jamshedpur. The school has invited Prof. G.Sagar, an eminent Mathematician from I.I.T Kanpur to deliver a lecture on Vedic Mathematics for the students of XI & XII (Sc.) on the occasion of International year of Mathematics. Draft a  notice supplying suitable information and details of the programme within 50 words.

Ans. 

 

SACRED HEART SCHOOL, JAMSHEDPUR

 

NOTICE

 

14th September 2014

 

Lecture on Vedic Mathematics

 

All the Students of Class XI & XII (SC) are here by informed that a  Lecture on Vedic Mathematics by Prof. G Sagar  from I.I.T Kanpur, will be organised by the School, on occasion of International year of Mathematics on 28th September, at 9-am in the Library Hall.  All are requested to be present in the hall before 15 minutes. For more details contact the undersigned

 

Romita

(ROMITA)

C.C.A Capt.

 

 

PRACTICE TASK

Write each of these notices in not more than 50 words.

 

  1. You are Mrs Arti Dua, Activity Incharge, of Great Field School. Write a notice asking the senior members of the Activity Club to attend a meeting to plan the fete that the school proposes to organize the following month.

 

  1. You are secretary, Blood Bank Society. Write a notice which you wish to put up in all the colleges in your city, Gandhi Nagar, asking for volunteers for blood donation. Give the necessary information.

 

  1. You are Ritika Seth, a student of ABC High School. You have lost your silver Timex watch in the playground. Write a notice describing the watch and giving the other necessary details.

 

  1. You are Deepak Kumar, head of the Sports Cub in Prateek High School. Write a notice calling for a meeting of all class representatives to decide the events and itinerary for the forthcoming Annual Sports Meet. Ask them to be ready to give suggestions.

 

  1. You are Rashmi Sharma. You have found a large amount of foreign currency near the school bus stop. Write a notice informing your fellow students of the same and asking the claimant to approach you or the school office with details of the denominations of the lost money.

 

  1. Write a notice on behalf of the City Sports Centre, inviting participants for an Inter School Tennis Tournament. Give details of the trial matches to be held at City Sports Centre and the prize money to be awarded to the first three winners.

 

 

 

 

BIO-SKETCH

The students will be asked to attempt a Biographical Sketeh in upto 80 words. Some notes will be added on an individual’s life or achievements. Students are expected to expand these notes into a paragraph.

Characteristics of a well-written Biographical Sketch:

  • A proper heading/ title relevant content
  • The content should include all given hints
  • Important aspects should be highlighted with Accurate Expression
  • Fluency
  • Accuracy

Important Tips

  • A biographical sketch conveys information about a person’s life, achievements, likes/dislikes in a comprehensive manner.
  • A well-written biographical sketch shows a student’s skill of organisation of matter as well as language accuracy.

 

SAMPLE BIO-SKETCH

 

Q.1.

Given below is a profile of Dr. Manoj Kumar. Write a short bio-sketch of Dr. Manoj Kumar. You may take the help of the clues given below :

Age                        36 years, robust physique

Height/Weight      172 cm, 85 kg, pleasing and dynamic personality

Family.                       has a joint agriculturists’ family, has five brothers and two sisters,

                             brothers well settled, leading a contented life

Hobbies                 riding, dancing

Professional          teaches political science in a girls college

Responsibilities    handles administrative responsibilities competently

Prominence          liked by all due to benevolent nature, cooperative attitude,                   clarityof thought

Ans.

Bio-sketch of Dr. Manoj Kumar

Dr. Manoj Kumar is a young man of 36 years of age. He is 172 cm tall having a robust physique and a pleasing and dynamic personality. He has five brothers and two sisters. His family is basically an agri­culturists’ family. He has a son and daughter, who study in a public school. Dr. Manoj has a Ph.D in political science and teaches in a girls college. He leads a very regulated and disciplined life. He loves to work to the maximum. Because of his administrative acumen, he manages the administrative duties of the college with a grand success. His hobbies include riding and dancing. He is always ready to help the needy because of his benevolent and kind nature. Naturally he is loved by all because of his understanding, pragmatism, clarity of thought and cooperative nature.

 

Q.2.

 

Given below is some details of the famous Booker prize winner and social activist Arundhati Roy. Using these details write a short description of her for your school magazine.

 

Arundhati Roy:-    Famous Indian writer

Born: –                  24th November 1961

Mother: –              Keralite

Father –                Bengali

Childhood–            In Aynaman in kerala.

Schooling:             Corpus Christi, Kottayam &  then  Lawrence School, Tamilnadu.

Higher Studies: – At the School of  Planning & Architecture, New Delhi

Contribution:        Written various social and environmental & political issues,

2 Screen plays and several collections of Essays. She also played the role of a village girl in the award winning movies Massey sahib.

1997 –                  Won the Booker prize for her novel The God of small things.

                             Now a Social activist and a freelance columnist.

 

Ans .

 

Arundhati Roy, the famous Indian writer was born on 24th  November, 1961. Her father is a Bengali & Mother a Keralite. Her childhood was spent in  Aynaman in Kerala. She did her schooling  from Corpus Christi, Kottayan and then from Lawrence School, Tamilnadu. She completed her higher studies at the school of Planning & Architecture New Delhi,. She has written on various social, Environmental & Political issues. She has also written two Screenplays &several collection of essays. She also played the role of a village girl in the award winning movie Massey sahib. In 1997, she won the Booker prize for her novel “The God of Small Things”. Now she is a social activist and a freelance columnist.

 

 

PRACTICE TASK

 

  1. Given below is a profile of Mr Suresh Kumar, your Mathematics teacher. Write a short biosketchof Mr. Suresh Kumar. You may take the help of the clues given below: (Word limit: 80)
  • age-40 years
  • height/weight/appearance-6 ft 1″, 75 kg, smart, good-looking
  • education-M.A. (Mathematics): Agra University. B.Ed: M.D. University Rohtak.
  • experience-1 5 years
  • duties-teaching mathematics, class teacher, organises Mathematics Olympiad
  • why I like/dislike him-painstaking, thorough, explains in detail, solves our problem, easily available, patient listener, cheerful-friendly approach.
  1. Your school is organising an Inter School Debate competition. You have invited famous authorAnamika Rao to judge the Debate Competition. You are a Senior Prefect and a member ofthe school Debate Club. You have been asked to write an introduction in about 80 words forthe author as part of the opening speech for the competition. You are Artij Arun from Mount.

High School.

  • Name: Anamika Rao
  • Age: 39
  • Genre: Fiction
  • Qualifications: BA English from Oxford University
  • University. MA English from University of London
  • Books Written: ‘Of Life and Love’ 1998; Gods and Humans 1999; Beautiful Lives 2004; various others
  • Awards: Bookworm Prize 1998; Writers Guild Award 2003 among others
  • Magazines: Home Life/National News/Times
  1. A new dance teacher has recently joined your school. You are a Senior Prefect and have been askedto write/speak about 80 words on introduction of your new teacher during the Morning Assemblyin school. You are Gauri/Gaurav from Rising High School.
  • Name: Radha Patel
  • Age: 34
  • Specialisation: Bharatnatyam
  • Qualifications: BA and MA in dance from Bharatiya Dance Institute, Bhopal
  • Solo Performances: ‘Natya Sandhya’-Delhi 2002; ‘Illusions’ – Bhopal in 2007/ among others
  • Group Performances: ‘Lights and Movement’ – a show depicting dance forms from various countries held at HABITAT Centre 2007 among others
  • Awards: Rashtriya Nritya Award 2004; Dancers Club Award.
  • Schools previously Taught in : Sant Kabir School Surat, Gujrat

 

  1. You are having an annual art exhibition in your school. As the Chief Editor of the school magazineyou are expected to give a writeup about the chief guest who is an artist of international fame.

Using the information provided below write a Biosketch of the artist in about 80 words. You areRachit/Rehana of Sagar Global School.

  • Name: Prashant Murthy
  • Age: 34
  • Qualifications: BA-Fine Arts From Baroda, MA-Fine Arts from Delhi College of Arts. Degree in Art Appreciation from University of Scotland
  • Number of Solo Exhibitions : Jahangir Art Gallery 1990/ Mudra Art Gallery 1991 / Rhea Art Gallery 1994/1995/ Habitat Centre 1996/ 1997. Various others
  • Number of Joint Exhibitions: ‘Impressions’ – Vadhera Art Gallery 1999/ ‘Voices and Visions’ Bharal Bhawan Gallery 2000. ‘face to face’Jahangir Art Gallery 2009
  • Awards: Golden Artist Award 1996/ President’s Award 1998
  • (Grants and Fellowships: British Council Art fellowship 1991. Stanford Art Association Grant 2007
  • Major Influences in His/Her Art Work : Influenced by the paintings of MF. Hussain and Raza, both of whom are modern painters

 

 

INFORMAL LETTER

POINTS TO REMEMBER

 

  • These are personal letterswritten to parents, friends, relatives.
  • Correct format should be used.
  • Letter should havecontent, fluency, accuracy, brevity.
  • It should be divided into at least three paragraphs – introduction, body of the letter, conclusion.
  • Adhere to the word limit.

 

Format

 

Sender’s address

 

…………………….

…………………….

 

Date

…………………….

Salutation

 

Introductory paragraph

 

Body of the letter

 

Concluding paragraph

 

Complementary close

Name

Sample Personal Letter

Q.1.

Write a letter to your friend Swaminathan telling him why you have taken up Commerce instead of Science.

 

41 Rajpur Road

Dehradun -425421

 

3 April 2014

 

My dear Swami

 

You will be glad to know that I have got admission in D.A.V. Public School.

 

 

You may be surprised to know that I have taken up Commerce and not Science. The reasons are obvious. Commerce includes all forms of trade and all services which help trade like banking, insurance, transport, advertising etc. In the modern time commerce is acquiring greater and greater importance. Without the help of commerce, industrialization of the country is impossible, because commerce is concerned with distribution of goods produced by industries. Commerce teaches the knowledge ofAccounts and Taxation which is indispensable even for a lay man. Moreover, there is a great competition in the field of Science, and this is not to my taste. In other words, I have no aptitude for Science. And, therefore, I have decided to take up Commerce.

I hope you will appreciate my choice of subject.

 

Yours lovingly

Srinivasam

 

PRACTICE TASK

 

  1. You are Shweta of Scindia School Gwalior, You have just returned from an educational tour to Rajasthan. Write a letter to your friend Harsh residing in Delhi about your experiences and your enjoyment.

 

  1. Tushar’s sister is in a hostel and is sick for the past two weeks. Tushar comes to know about this and is disturbed. He writes a letter to his sister advising her to take proper and healthy diet so that she gets well soon.

 

  1. You are Mohini, who has been selected for a job in U.K. You have never been to any foreign country so you are a little nervous. Write a letter to your friend residing in London to give you some information about the place.

 

  1. Recently you went to a remote village for the Adult Literacy Mission. You enjoyed teaching the men and women of the village. You were thrilled to see their curiosity to learn. Write a letter to your mother requesting her to teach the illiterate maid in your house.

 

  1. You have been admitted to a medical college in Pune. You have recently read in the newspaper about incidents of ragging and eve-teasing. You are afraid to join the college. Write a letter to your father to accompany you to your college. This will give you strength and moral support. Your name is Adarsh.

 

 

DATA INTERPRETATION

Guidelines

Our age is the age of computers. Data is pouring fast in print media and other visual media in very attractive form. Proper interpretation of data is essential because only then can we analyse and appreciate the importance and relevance of the problem or the issue.

The students will be asked to transcode the information from a diagram, map, table, chart or illustration into a short paragraph of about 80 words. The students will be provided some visual stimulus as data. Students are expected to interpret the data.

 

Characteristics of a well-written Data Interpretation.

  1. A proper heading/title
  2. Relevant content
    • The content should include all given points
    • Important points of data should be highlighted
    • Comparison/contrasts should be properly focussed

iii.   Accurate Expression

  1. Fluency

v.Accuracy

Important Tips

  • Data Interpretation aims at transcoding information from a diagram, map, table, chart or illustration into a short paragraph.
  • It aims at assessing a student’s skill of expressing ideas in clear and grammatically correct English.

SAMPLEDATA INTERPRETATION

Q.1.

Study the following pie chart carefully which lists various sectors responsible for gas emissions and their share  in global warming. Using this information write a passage in about 80 words on Global warming.

 

Transport (27%)

 

Others (1%)

 

Residential (6%)

 

Commercial (7%)

 

Agriculture (7%)

 

Industry (19%)

 

Electricity Growth (33%)

 

 

Ans.

GLOBAL WARMING

 

The pie chart above shows various sectors responsible for gas emissions and their share in creating global warming. The most important factor that contributes to this menace is the transport which emits 27% of poisonous gases, 33% of this responsibility goes to electricity growth. The industries have 19% and Agriculture has 7% share in creating global warming.7% of responsibility is shared by the commercial establishments and 6% by residence to increase the deadly problem. There are other sources which takes 1% share to contribute to this menace of gas emissions and global warming thereafter. The above sectors should try their best to reduce the rate of pollution and global warming before it is too  late.

 

PRACTICE TASK

  1. Given below is a table showing the sales record of a shop:
Month Indian Film Music Indian Classical Music Western Music
  (figures are in thousands)
Records Cassettes Records Cassettes Records Cassettes
January 180 150 20 05 30 10
February 200 150 15 10 40 10
March 190 170 40 10 50 15

Write a short paragraph in not more than 80 words interpreting the above data.

 

 

  1. Look at this visual from a report on teenage problems. On the basis of the data, write a short paragraph in not more than 80 words on the problems and stresses faced by teenagers today.

 

 

  1. A survey was conducted to find out how teenagers spend their free time. The trends are depicted in the pie charts given below. Write paragraph on ‘Teenagers and Leisure Hours’.

 

 

 

 

 

E-MAIL

Guidelines

E-mail or electronic mail is one of the most popular uses of the Internet today. The advantages of e-mail over regular mail is that it is free, it is delivered almost instantly and all types of files can be attached to the messages.

E-mail is basically like writing a normal letter and sending it through the post office (“snail mail”), only it is fast, inexpensive and convenient.

While sending an e-mail, you just follow some basic rules that apply to regular mail. You need to have an address, a message, perhaps an attachment, -but no envelope or stamps. Your message will be sent and received by the click of a button in a few seconds.

Parts of E-mail

(a)      E-mail address: E-mail address has two parts-your user ID (usually your name) and the identity of your e-mail service. These two parts are separated by the symbol @.

For example, the e-mail address of Saraswati House is written as : sa [email protected]

This would be spoken as saraswatihouse at hotmail.com

Here (i) saraswatihouse’ is the user ID for e-mail service, (ii) @ is an “at” sign.
It separates the person’s name from the name of the e-mail service being used, (iii) hotmail.com indicates the e-mail service being used and its location on the web.

First mention sender’s e-mail address with date and then the receiver’s e-mail address.

(b)      Subject: Write a line that describes what your message is about.

(c)      Message: There is no limit to an e-mail message. However, for examination purpose, the limit is 120 words.

(d)      E-mail Attachments: Any kind of file may be attached to the message.

Points to Remember

  • Be careful to spell the address right.
  • Use the lower and upper case letters correctly.
  • Press the SEND button when you have written your message.
  • DON’T TYPE IN ALL CAPS-This is interpreted as shouting.
  • Limit the use of multiple exclamation marks and question marks.
  • Remember this is non-verbal communication.
  • Style of writing is almost as important as the content.
  • E-mail should be in a box.

Emotions

  • Use emoticons (emotion icons) when trying to convey a tone of voice
  • Emoticons sometimes referred to as smileys, are keyboard symbols.
  • These help convey the tone, or emotion, of an online message.

 

  • Some Emoticons are given below:

🙂      happy                    😀     laughing

🙂        smile                     🙁      sad

:-e      disappointed                   >:-<    mad

🙂      wink                      :-]      indifferent

-:0      A baby                   😮      shocked, surprised

:-@     screaming             :->      devilish

:-/      hmmm

 

Acronyms

          When corresponding with friends, these acronyms can save time.

[These are well known to people who utilize newsgroups and mailing lists)

2L To late AAMOF As a matter of fact
AFAIK As far as I know B4N Bye for now
BRB Be right back BTW By the way
CMIIW Correct me if I’m wrong CUL See you later
FWIW For what it’s worth FYI For your information
IAC In any case IAE In any event
IKWUM I know what you mean IMHO In my humble opinion
KWIM Know what I mean IOW In other words
LOL Laughing out loud NBIF No basis in fact
OTOH On the other hand ROTFL Rolling on the floor laughing
RSN Real soon now RTM Read the manual

 

Format of E-mail

 

  • Sender’s E-mail address
  • Receiver’s E-mail address
  • Date
  • Subject
  • Message
  • Attachments (if any)

 

 

Solved EMAIL

 

Q.1

 

You have been participating in an Environment Protection Week. Write an
e-mail to your cousin describing your experience in minimum 120 words. You may take help from the following points:

  • treatment of garbage
  • no garbage heaps
  • different dustbins for garbage
  • one for biodegradable
  • other for non-biodegradable
  • separating at source
  • uses of separate bins

Ans.

 

From : [email protected]

______________________________________________________________________________________

To : [email protected]

______________________________________________________________________________________

Date :10 October, 2014

______________________________________________________________________________________

Subject :Garbage Treatment

______________________________________________________________________________________

 

 

Dear Shruti

 

Attending the Environment Protection Week was an eye-opener. It made us aware how to keep the environment neat and clean as well as how to turn garbage to gold. There would be no garbage heaps or landfills spoiling the beauty of landscape. A simple process, adopted at source i.e. in homes/offices/hotels/restaurants etc. can solve the problem. There is nothing extraordinary or innovative about it. We have to stick to basics. Two different dustbins have to be kept for collecting garbage. One of these is for bio-degradable waste and the other is for non-biodegradable garbage. The biodegradable waste can be turned to manure and used for increasing the yield of fields. Various elements in non-biodegradable waste are separated and recycled. Thus treatment of garbage can produce healthy and encouraging results.

 

Yours lovingly

Sonali

 

 

 

PRACTICE TASK

 

  1. You are Sudip, a student of class – X. You recently visitted the National Museum at Delhi. Write an e-mail to vour friend Susan who is in Denmark about what you saw in the museum in about 120 words.

 

  1. You are Joseph. You recently visited Gir forest in Gujarat. These are famous for the Asiatic lions. You were lucky to have spotted a mother and cub. Write an
    e-mail to your friend Sudha about your visit.

 

  1. You recently attended ‘Tourism Week’ organised by Rajasthan Tourism Development Corporation. The highlight of the event was information on the various tourist places to visit in Rajasthan, the cuisine of the state art and crafts of the state. Write an e-mail to your friends Shreya telling her about Rajasthan.

 

  1. The IPL cricket has not only thrown open new avenues for cricketers but it is enjoyed by the public as well. The cricket crazy people of India watch these matches with passion. You areNandita, a class X student. You however feel it is a sheet waste of money. You decide to share your views through an e-mail with your friend Rajat who is a fan of the game. Write the e-mail in about 120 words.

 

 

STORY WRITING

 

Strategies for writing a story

 

  • In order to write a story keep the following points in mind
  • First create the outline of the story.
  • There are many ways to write a story but it should basically have the following elements :
  • Setting of the story
  • Characters – ensure that you give names to the main characters
  • Problem – the characters should encounter a problem but don’t make the

problem too complex to be solved.

  • Solution – the characters should find a solution to their problem
  • Reaction – would be how the characters feel once the problem has been solved.

 

  • This would generally be the conclusion of the story.
    • Avoid repetition of incidents and words. Do not stretch one point too far. Keep the story moving. The reader should be keen to read on.
    • Make the plot logical and realistic unless you have been asked to write a fantasy.
    • Keep in mind that grammatical accuracy and correct spellings are essential.
    • Avoid the use of slang and too much dialogue.
    • Use sequence markers to keep the plot cohesive.
    • Give the story a suitable title.
    • Word Limit : 120 words.

Sample Story Writing

 

Q.1.

 

Write a story about a brave dog.

 

Ans.

Sangeeta and Ramakrishnan’s family consisted of three children and adog. The oldest among the children was Dinakaran, a seven-year-old boy.The dog, Selvakumar, was yellow and scruffy. They had always livedalong the coast in a fishing village. It was just north of Pondicherry,a former French colony in South India. The morning of 26 December 2004began like most others, with sunny skies and a very cool breeze.Sangeeta’s husband, Ramakrishnan, had just returned from his earlymorning fishing. Suddenly, they heard a strange noise comingfrom the sea. Ramakrishnan went to have a look. He ran up to theroof and saw the waves. He shouted down to Sangeta. “There’s no time; run away, run with the children!”That’s when Sangeeta made her agonizing choice. She couldn’t carry all three children.“Dinakaran is somewhat older than the other two. I knew he would be able to run, so I grabbed the other two and asked him to run too,” Sangeeta explained later. However, Dinakaran ran towards the family hut. It was then that the dog ducked into the hut. Nipping and nudging, he carried Dinakaran up the hill to safety. Dinakaran says the dog saved his life.

 

PRACTICE TASKS

 

  1. Build a story based on the following outline.
  2. Bag lost in a train by a traveller
  3. Bag found by an attendant
  4. Attendant rewarded

 

  1. Write a story that illustrates the expression ‘Look before you leap’.

 

 

 

SPEECH WRITING

 

A speech is an effective way of communicating a message to a large audience. It is one of the ways of spreading awareness regarding social issues or information about other important issues.

 

Tips for writing an effective speech

  • Read the question carefully.
  • Focus on the part of the question/title that needs to be addressed.
  • Interpret the data given, if any.
  • Use a conversational style of writing.
  • Do not use slang.
  • Use polite, grammatically accurate language.
  • Avoid repetition of facts and figures.
  • Add variety to your vocabulary/expression.

Sample Speech

 

Q.1

You are the head boy of your school. You wish to encourage the students to help in eradicating the social evil ofchild labour. Write a speech in about 120 words, which you would like to deliver in the morning assembly.

 

Ans.

 

Good morning respected Principal, teachers and dear friends.

 

Today I would like to speak on one of the major social evils in our society. In the book ‘Lost Spring: Stories of Stolen Childhood’, Anees Jung has beautifully brought out the apathy of the privileged class towards the slum dwellers and communities that are pursuing traditional trades like bangle-making. The government has laid down laws against child labour but so often we see little children working and we turn a blind eye to it. Many factory owners lure children into working for very low wages. Poverty and corruption have further augmented this problem.

 

Could we as students of a school and as the future leaders of the country, do something about this? Yes, we could. Each one of us could take the responsibility of teaching one child. We could support the government and the NGOs in their endeavour to spread awareness regarding the drawbacks of child labour. The media too has a role to play. Documentary films and advertisements could go a long way in eradicating this evil and bringing the children’s stolen childhood.

 

I would like to conclude by saying that we should join hands to fulfil the mission of eradicating child labour from our country. Only then can we achieve what seems to be unattainable. The rate of crime and violence would come down. Our country would have a higher rate of literacy and subsequently, our economy would get an impetus. So, let’s restore the rights of the underprivileged children and build a better future for them and ourselves.

 

Thank you.

 

 

PRACTICE TASK

 

  1. A study by scientists has revealed that a majority of patients suffering from heart disease and obesity are those who are leading a sedentary life. They spend long hours at their desk and do not find the time or opportunity to either exercise or play sports. You are the head girl of your school. Write a speech which you would like to deliver in the school assembly, emphasizing the importance of an active and healthy life.

 

  1. On 20 January 2007 Bhabha Atomic Research Centre completed 50 years of its existence. Its contribution in nuclear power programmes, helping cancer patients, protecting food items from rottening and even developing new crop varieties has helped India progress. Write a speech for your school assembly, highlighting the contribution of science and technology towards the development of our nation.

 

  1. Sometimes consuming street food is unavoidable. It is often tempting and economical. As the head boy ofyour school, you would like to spread awareness regarding the food sold in the streets. Write a speech for your school assembly warning the students against the hazards of consuming contaminated food and suggest precautions that should be taken.

You may use the suggestions given below :

  • best consumed hot and fresh
  • should be covered
  • only permitted food colours used
  • location – away from garbage and open drains
  1. The media is a great source for spreading awareness regarding health, social and environmental problems and other areas related to our lives. Write a speech highlighting the positive impact of the media on the life of the common man.

The following points may help you.

  • provides information
  • civic problems can be highlighted and solutions suggested
  • awards announced through the media – provide encouragement
  • serves as a forum for discussion of topical events
  1. The rate of crime and violence is increasing each day. As aconcerned citizen, you wish to appeal to the people to overcome communallism, extreme desire for materialism and intolerance. Write a speech encouraging your fellow students to look at the broader perspective of the impact of vices on our life. Emphasize the effect of violence and also give some suggestions to overcome this evil.

 

 

 

FORMAL LETTERS

 

Letters to government officials, business houses, customers, editors, colleagues and acquaintances fall under the category of formal letters. As the term itself indicates, a formal letter is written in a formal style i.e. the language and tone of the letter as formal, impersonal or objective, therefore, no informal expressions, greetings or contracted forms of words are used. The language is straight forward and to the point. It may be strongly worded if need be but never impolite. You will find the following guidelines helpful in writing formal letters.

 

To the Officials   :       Begin by identifying yourself. Then clearly state the purpose of       writing.

Give details of what you want to do or want to be done e.g.                              applying for a newelectricity connection.

Make a request for appropriate/necessary action.

 

To the Editors    :       You may begin with a reference to recent developments, new

events etc.

Establish a context for writing.

Give details of the topic e.g. the problem of beggary, its causes                                          and effects andpossible solution.

Do not ask the Editor to solve your problems. He can only give voice to your views, concerns, complaints, suggestions, criticism, appreciation. He may also be approache for complaints against erring offiials, departments, business houses etc.

 

To Boss &          :       Show due respect but do not flatter.

Colleagues                 Clearly state the purpose of writing e.g. asking for a raise,                                 seeking action on aproposalor promise, making enquiries etc.

 

Acquaintances   :       Use polite language and pleasant tone.

Always be very polite (even if you have to complain).

 

To Business       :       Keep the tone friendly and pleasant.

Houses                      Do quote reference numbers, order numbers, cheque / demand

Customers                 draft numbers, filenumbers, code or ID numbers etc.

Convey your message in a brief and straight forward manner.

Give more importance to the reader.

 

We write formal letters to:

  • apply for job applications.
  • get our problems addressed/seek redressal to them
  • inform others
  • persuade others
  • express our views on public issues.
  • appeal to others
  • warn someone
  • complain against someone
  • make enquiries.
  • send replies.
  • book/supply orders
  • send /ask for payments/discounts/brochures of information etc.
  • sell products.

Points to Remember

 

  • In Formal Letters there should be no use of informal expressions, greetings or contracted forms of words.
  • Language is straightforward and to the point.
  • It may be strongly worded if required but never impolite.
  • The heading includes the writer’s address and date. These are written at the top left hand corner of the letter.
  • The date should be written below the address.
  • Name/Designation and complete address of the recipient s written below the date line from the left side margin.
  • The salutation is written just below the date with a double space separating the two. Its choice depends upon the relation and intimacy the writer has with the addressee. For Professional Persons: Dear/Respected Sir/ Ma’am
  • The complimentary close/ subscription is a polite way of ending a letter. The expression used must match the salutation.
  • The signature is written just below the subscription.
  • Word limit : 120-150 words.

Common Mistakes Usually Made by the Students

 

  • Writer’s address should precede the receiver’s address.
  • The editor should not be asked to solve your problems. He can only give voice to your views, concerns, complaints, suggestions, criticism and appreciation.
  • Show due respect but do not flatter.
  • Do not be disrespectful; always be very polite (even if you have to complain).
  • Do not forget to offer plausible suggestions to the problem.

 

 

Format:

 

OWN ADDRESS

_______________________________

_______________________________

 

DATE

 

RECEIVER’S ADDRESS

_______________________________

_______________________________

_______________________________

_______________________________

 

SALUTATION (RESPECTED SIR/SIR)

 

SUB:

 

CONTENT-   1ST PARA- INTRODUCTION

2ND PARA- EXPANSION

3RD PARA- SUMMARISE

4TH PARA- CONCLUSION

 

COMPLIMENTARY CLOSE

SUBSCRIPTION

SIGNATURE

FULL NAME

 

 

Sample Letter

 

Q.1

 

Write a letter to the editor expressing your views on the growing corruption levels in the country. Also mention the ways by which this growing menace can be tackled.

 

L-119

Scc-56 Gurgaon

Haryana

 

29th October 2014

 

The Editor

The Times of India

Bahadur Shah Zafar Marg

New -Delhi-110048

 

 

Sir

 

Subject : Growing Corruption Levels in the Country

 

Through the columns of your esteemed daily, I would like to draw your kind attention to the growing cases of corruption in our country. It would not be wrong to say that corruption has seeped into the very fabric of our society.

 

Every day we hear politicians taking bribe or scams being unearthed by the media. It is sad to witness the tremendous strides our country has taken and the shallowness of our system. As a responsible citizen of the country myself, I feel some of us are completely unaffected by the state of affairs going on in our country, whether it is the CWG lapses or the 2G scam.

 

In order to bring a change in the system I sincerely feel that the Government should take stricter actions against the perpetrators of law and punish them severely. People should raise their voice against loopholes in the system. There should be public forums, symposiums organized where we can express our opinions. It is really the need of the hour and hope some prompt action shall be taken for the same.

 

Thanking you

 

Yours sincerely

Abhinav Sharma

 

 

 

PRACTICE TASK

 

  1. Write a letter to the editor of a newspaper appealing to the general public for contribution to the Chief Minister’s Drought Relief Fund. (Word Limit: 120 words).

 

  1. Write a letter to the editor of a newspaper expressing your disapproval of the so called inaugural functions which cause so much national waste. Give instances of such waste from your observation.

 

  1. Suppose you are an educationist and feel strongly about the flaws in the existing system of examination. You feel that the system neglects the talents and skills of students and is restricted to classroom teaching. Write a letter to the editor of an English newspaper making a case for education beyond classroom teaching. Sign yourself as Dr. Manoj Lamba.

 

  1. Write a letter to the Editor of the daily, “The Tribune”, about rash and reckless driving, suggesting preventive measures. You are Navin/Neha living at B-47, Friends Colony, and New Delhi.

 

  1. Last year a fifty-year old man died of cardiac arrest while taking a ride on a swing in an amusement park. Write a letter to the Editor suggesting some safety measures which park owners must observe before issuing tickets. You are Parul/Parijat, living at 15, Subhash Nagar, Jaipur.

 

 

ARTICLE WRITING

 

The purpose of writing an articleis to present information on a variety of themes in a long and sustained piece of writing.

 

Format

 

  1. HEADING :   should be eye catching; encapsulating the central theme. Byline — the name of the author follows immediately.
  2. INTRODUCTION

Theopening paragraph must

  • tell what the article is about
  • catch attention
  • arouse interest
  • limit and control what you plan to discuss in your article
  • have clear and precise language; may even use a definition or quotation
  1. DEVELOPING CAUSE AND EFFECT RELATIONSHIP
  • use facts to support your claims
  • give examples to support your views
  • present arguments in a coherent, logical and convincing manner

 

  1. COMPARISON   AND CONTRAST
  • give views contrary to yours
  • argue as to why your views are better
  1. CONCLUSION
  • summing up including a consolidation of ideas
  • offering suggestions/ measures to improve the situation
  • personal observations and predictions

Value Points

  • Give a suitable heading
  • Do not attempt to write about every single piece of information- select relevant information judiciously
  • Write in an appropriate style and format

Sample Article

 

Q.1

 

Shyam is a boy who has survived major accidents; it makes you wonder about the cause of accidents in the Indian cities. You decide to write an article for the school magazine titled ‘Driving India Crazy’. Taking ideas from the data below and your own ideas write the article.

 

 

Number of persons killed in cities (yearly)

 

Ahmedabad 22
Bangalore 659
Chennai 692
Delhi 1989
Mumbai 449
Kochi 148

 

Ans.

Driving India Crazy

 

Indian roads can aptly be called the ‘Killing Roads”. It is difficult to believe that Shyam , a young boy has survived two accidents, both caused by the negligence of others. If you study the statistics, it can be seen that Delhi leads in the number of persons killed on the road, a huge 1989. Every day we read reports of people dying, getting hurt or maimed on the killer roads. It, therefore becomes imperative to study the reasons for this phenomenon.

 

Underage driving, granting of licence to unqualified and untrained people is one main reason. But on the other hand, there is also a total lack of road   safety rules among the general public. Rash driv­ing amongst the youngsters is a matter of pride. Hardly anyone adheres to the road rules. Jumping   red lights is a daily occurrence, as the Traffic Police looks the other way.

 

Therefore, it is no wonder to see the chaos and the increase in road accidents. Even in the disciplined city of Kochi, 148 people died in road accidents. Of course, there is an immediate need to stop this awful loss of life.

 

The answer lies in being responsible. The public and the Traffic Police need to be vigilant. Those flaunting traffic rules must be dealt promptly and strictly. Let us stop this wanton loss of life on roads. Let us make our roads   safe by just following simple rules. Remember that “Speed thrills but kills”.

 

 

PRACTICE TASK

 

  1. You have read the following news item in your local newspaper and decided to write an article for your school magazine on global warming. You are Anita/Avinash

As pollution increases in major cities across the world, the average temperature of the world has gone up by almost 3ºC   in the last 25 years. This is seen as having   far reaching consequenc­es for the environment.

 

  1. You overheard the following conversation:

RAHUL       : Where have you planned to go during the summer holidays?

JEEVAN       : I wish I could go boating on Dal Lake.

RAHUL        : Is it safe   to go to Kashmir now? Don’t you hear a lot about                         terrorism in the news­papers?

 

On hearing this dialogue you decide to write an article in the local daily on the need to provide security at tourist spots and suggest ways and means by which such spots could be made more attractive to tourists. Using ideas from the unit on “Travel and Tourism” along with your own ideas, write an article in about
150 words.

 

  1. You decide to write an article for the health magazine showingyour concern about the deadly disease. Also give suggestions to take measures to prevent it fromspreading. Diabetes is a disease on the rise throughout the world , especially in the developed anddeveloping countries. As a student, you are concerned about the rise in diabetes deaths and also therisk of blindness, kidney failure, amputations and the heart failures faced by people who suffer fromdiabetes. Write an article in about 150 words.

 

  1. Shobita recently came across several news reports regarding the acute water shortage. On the occasion of World Water Day, she decides to express her views on the grave situation, the causes behind the crises and also the suggestions to improve it in an article to be published in a magazine.

 

  1. Arjun is fascinated by the extensive use of gadgets in everyday life. He decides to write an article for his school magazine expressing his views regarding the way modern gadgets have revolutionized our life and the need for man to act responsibly in the use of gadgets. Using your own ideas and the ideas from the unit on ‘Science’ wtite the article in about 120-150words.

 

 

 

DEBATE WRITING

 

A Debate is a formal method of interactive and representational argument. The purpose of a debate is to convey information orally to a large gathering of people, forcefully and convincingly and to convert the listen­ers to the speaker’s point of view.

 

Points to Remember

  • A catchy phrase or statement/quotation is always preferred at the opening. This enhances the impact of the content.
  • A formal greeting and self-introduction is essential.
  • Content should be supported with arguments and counterarguments to give it the desired impact.

Format

Title /Heading

1st Para- Opening statement/ Quotation+ Formal Greeting+Self

Introduction+Statement of Topic (Motion- For or Against)

2nd Para- Expansion of the concerned topic with supporting facts

3rd Para- Summarize+ reiterate your stand

4th Para- Conclusion

 

Sample Debate

 

Q.1.

 

Write a debate about “Discipline in schools-A dying trend”.

 

Ans.

Discipline in schools-A dying trend

 

A very good morning to respected Principal Madam, Vice-Principal Madam, teachers and my dear friends. Today I Sayesha kumar stand before you to express my views against the motion-Discipline in schools a dying trend.

 

Asa student myself, I truly believe that the most amazing part of my school is the discipline exerted on us. How can we then say-that it is a dying trend. What happened to India garnering respect the world over for its unique study pattern and the fact that Indian teachers commanded more respect from their students?

 

India has been famous for its guru-shishya parampara which basically talks about the unique bond that the teacher has with his pupil. Such a relationship is fostered on mutual trust, understanding and reverence that the pupil pays to his teacher.

 

Our very curriculum is based on disciplined parameters. We have fixed periods for teaching, select subject options to choose from and our examination system is also disciplined in every sense of the word. Hence I would like to conclude by saying that discipline in schools is certainly not a dying trend.

 

Thank you

 

PRACTICE QUESTIONS

 

  1. You have participated in an Inter House debate contest in your school on ‘Hindi should be the medium of instruction in schools in India’. Write your debate in not more than 150 words.

 

  1. You are participating in an Inter Class debate Competition on ‘Peer Pressure is not always beneficial.’ You may take ideas from MCB Unit on ‘Children’. You are Sanjiv/Saroj of The Little Genius Debate Society. Write in minimum 150 words.

 

 

  1. You are taking part in an Inter School Debate Contest. The topic of the debate is, ‘Animal performances should be prohibited in circuses.’ Write for or against the topic in a minimum of 150 words.

 

  1. Schools have instituted all kinds of prizes for students. You have to participate in an Inter-House Debate Contest, the topic being, ‘Schools should end the practice of giving grades.’ Write your debate in not less than 150 words.

 

REPORTING AN EVENT

 

(Events and Experiences)

 

A Report is a factual description of an event or a happening. It may also give an analytical analysis of an event. A good report is always based on first hand information about the incident/ event / happening. A reportcan be a lengthy document or a short piece of description, depending upon the nature of the report. There are many types of reports, but we will confine ourselves to newspaper reports describing events / incidents / happenings / functions and business reports.

 

Events / Functions

 

  • Name of the event
  • Situation / Occasion
  • Name of the Chief Guest / Name of the Person presiding over the function
  • Time, Date, Place
  • Gist of the Chief Guests’ speech
  • Description of the event / function
  • Something specific about the event/ function
  • Prize Distribution/ or any other ceremony performed by the Chief Guest
  • Thanks by one of the organisers
  • The writer’s analytical comment on the quality/ merit of the programme

 

Accidents

 

  • Cause of the accident
  • Where (Place)
  • When (appr. Time)
  • How (reason)
  • Vehicles involved in the accident
  • People’s help / Rescue and relief operation
  • Casualities if any
  • Persons injured
  • First aid
  • Inquiry by the government
  • Announcement by the government about grants to the next kin of the killed and the injured.
  • Analytical comments by the writer.

 

 

Format of a Formal Report

 

Heading

(By : Name, Designation)

Place, Date  :

 

Body of the Report:

  • Highlighting the subject / event
  • Contents of the report
  • Winding up the report, with recommendations and action to be taken.
  • Signature with Name and Designation.

Report and Factual Description

 

Points to Remember

 

For Newsreport

  • Give a heading.
  • Be sure to include all relevant information.
  • Newspaper reports should have headline, date, place of origin of news, name of reporter.

 

For Factual Description

  • Be sure to be objective.
  • Presentation and organisation of details should be systematic and logical.
  • Title and reporter’s name (date and place – optional)
  • Content should contain following points
  • When, Where
  • Why, What

Sample Report

Q.1.

As a TOI correspondent, write a short report on the inaugural function of Jeevan Rakshak Hospital and Heart Institute at Dayanand Vihar for your newspaper.

 

Hospital Inaugurated

TOI correspondent

 

New Delhi, June 5 : Jeevan Rakshak Hospital and Heart Institute, Dayanand Vihar was finally inaugurated by Mrs. Sheila Dikshit, the Chief Minister of Delhi, on Wednesday, June 7, 2006. The 200-bedded institute has been upgraded with the latest ultra modern and top-of-the-line machines, besides enhanced infrastructure to support exigencies. Addressing the gathering, Dr.Dheer, the Medical Superintendent, expressed his satisfaction with the arrangements and facilities provided at the institute and reinforced the need for total commitment of the medical staff and the other employees towards their noble profession. Leaflets were distributed, highlighting a brief history of the institute, facilities and the treatment available therein. The Institute is constructed at the cost of Rs. 20.18 crores and is spread over an area of 9,120 sq. metres.

PRACTICE QUESTIONS

 

  1. Your school has organised an Exhibition-cum-Sale of the items made by the students in their work- experienceclasses. You had an excellent and overwhelming response from the parents and the visitors. The proceeds ofthe sale have been donated by your school in a function to ‘Helpline India’, an organisation for supportingthe orphans. Make a report of the same in 125 words to be published in your school bulletin/ magazine.

 

  1. You are Kiran Malhotra, a staff reporter of The Hindustan Times’, Lucknow. You have witnessed a roadaccident involving a lorry and a Maruti Omni in Hazratganj, Lucknow. Write a report, giving the details of number of people injured and the extent of damage caused to the colliding vehicles in not more than 125 words for your newspaper.

 

  1. As the reporter of The Times of India’, New Delhi, write a report on the fire accident in the market-place, which you witnessed. You are Praveen/Parvathi. (Word limit: 125)

 

  1. Recently you attended a seminar on Pollution Control’ arranged by your school. In the seminar the partici­pants presented papers on how to control pollution, which was followed by a discussion session. Prepare a brief report in 125 words to be published in your school magazine. You are Pramod/Prita Head boy/Head girl of National Academy, Agra.

 

  1. You have received training in First Aid, as a member of Red Cross Society, arranged by your school. Prepare a report to be published in your school magazine about your training in about 125 words, Sign as Sagan/ Sagarika.

 

  1. Write a report as an eyewitness to the bus accident that occurred near B.M.C. Chowk, Ambala. Word Limit: 125 words

 

 

 

MESSAGE WRITING

 

A message is a short piece of informal writing. Students must be able to retrieve and interpret information in an appropriate style and format to convey information in a precise and clear manner.

The following points should be kept in mind while writing the message

  • Message must be placed in a box.
  • No address is to be given. Give only a brief salutation e.g. Ravi/Sir. [Do not use Dear].
  • Include all important details and adhere to the word limit.
  • Put your name/signature at the end of the written message.
  • Time and Date should be mentioned.
  • Always use Indirect Speech to write the message.
  • Write the name of the person for message.

Sample Message Writing

Q.1.

 

Read the following telephonic conversation between Radha and Mrs. Rawat.

 

Mrs. Rawat wants to talk to Radha’s mother. As her mother is not at home, she attends the call. Now it is time for Radha to go for her tuition. She writes the message for her mother related to her conversation with Mrs. Rawat. As Radha, write the message for her mother.

 

Mrs. Rawat: Hello! Is it 224567?

Radha: Yes, it is. May I know who is on the line?

Mrs. Rawat: Sure I’m Mrs. Rawat, your mother’s friend. Could I speak to her?

Radha: Good evening aunty, I’m afraid mummy isn’t at home. I’m Radha, her daughter.

Mrs. Rawat: Well, then could you please tell her that our much awaited yoga camp begins from today at 6 pm at the Community Centre, where the renowned experts will impart certain tips to ease BP and other diseases.

Radha:OK… I will convey the message to her.

Mrs. Rawat: Yes, tell her to reach the centre by 5.45pm. I shall join her there.

Radha: Fine. I’ll tell her.

 

Ans.

MESSAGE

12 July,2014                                                                                                 4.00 pm

 

Dear mother

 

Your friend Mrs. Rawat called up to say that your much awaited laughter Yoga Club Camp starts today at 6 pm at the Community Centre where the renowned experts will impart certain tips to ease BP and other diseases. She has asked you to reach the centre by 5.45 pm where she will join you. I am going for my tuition.

 

 

Radha

 

PRACTICE TASK

 

  1. Read the following conversation between Mr.Rajan and Sudha and write the message for her as she is leaving for her Music class. Write the message within 50 words.

 

Mr Rajan –  Hello, may I speak to Mr.Singh?

Sudha –       Sorry, Papa is not at home now. May I know who is speaking?

Mr.Rajan-    I am Mr. Rajan from the Telecom Department Kindly inform that your father’s  telephone bill for 3 months is pending till date.

Sudha –       All right. I shall inform my father as soon as he comes back.

Mr.Rajan –  Also inform him that if he fails to pay the bill this month the telephone line will be disconnected. He may also pay the bill online with nominal extra charges.

Sudha –       I am sorry sir. I will inform him and request him to pay the bill within 2 days.

Mr.Rajan –  Thank you

 

 

Signatory

 

 

 

FACTUAL DESCRIPTION

 

Q.1.

 

One of your family members fell sick and is admitted in a hospital for treatment. You made up your mind to know the state of the patient and visit the hospital. Your experience there was very shocking when you saw the poor plight of the patients and infrastructure. Describe your visit to the hospital in about 100 words.

  1. Use past Tense
  2. Include all the facts like unhygienic surroundings, stinking toilets, over        crowded rooms, stray dogs and cats inside, no security and low quality of         diet etc. within the word limit.
  3. No personal opinion while describing an Event.

 

Description of a hospital visited recently

Ans.

Last week, I visited Life Line hospital in Mirzapur. My first experience there was very shocking. The hospital surroundings were unhygienic with over flowing garbage bins and choked sewage. The toilets were not cleaned regularly and were stinking awfully. The rooms were over crowded with patients and some of them were lying on the floor. The patients were served with very low quality diet and impurifiedwater. Doctors were not regular in their visits. The stray dogs and cats were also sharing some of the beds with the patients increasing the risk of infection to the patients. As a whole the plight of the hospital beggared description.

 

Q.2.

 

A well dressed gentleman came to meet your father but he was not at home. The gentleman introduced himself as Mr.Samuel of St.Xaviers College, Ranchi. He said that he and your father had studied together in college and had not met for last 15 years. He promised to come again tomorrow. Give a description of this gentleman to your father to help him identify his old friend in 100 words.

 

Ans. 

Description of Mr; Samuel, my father’s classmate

 

Last evening a well-dressed gentleman came to our house  to visit my father. The gentleman introduced himself as Mr. Samuel of St.Xaviers College, Ranchi. He was of about fifty five years as he referred himself as a classmate of my father. He was quite healthy, of medium height  and of wheatish complexion. He had beady eyes, wavy hair and a sharp nose. He was bespectacled and was in a well fitting black suit. He had a mole on his left cheek. He was a well-mannered gentleman with a soft way of speaking with a sophisticated accent. He seemed to have a very sharp memory even after a gap of 15 years.

 

 

 

PRACTICE TASK

  1. Generally, schools in Delhi organize a Youth Parliament. You happened to attend such a parliament in your school. Draft a suitable paragraph on the above topic in about 100 words.
  2. Describe in your own words, the Motto of your school.
  3. Describe in about 100 words the process of applying for admission to a school.

4        Examination creates great terror and fear in the minds of the examinees. Preparing for an examination needs good planning, a strong will-power, labour and courage. Describe in about 100 words how did you prepare for today’s examination.

5.Describe your experience of witnessing the Republic Day Parade in not more than 100 words.

 

 

DIALOGUE WRITING

 

Guidelines for Dialogue Writing

 

Dialogue writing is a harder exercise than writing a biographical sketch or attempting data interpretation. It involves higher writing skills. The student has to provide not only a suitable sentence, an answer or a remark, but also keep the sense of the passage in mind i.e. the preceding and the following sentence.

 

The completed piece should read as an organized whole. The student is expected o present ideas in a coherent and concise manner.

Q.1.

 

Study the Dialogue between a shopkeeper and a Customer and complete the dialogue meaningfully within 80 words.

Customer: – Would you give me two kilos of Potatoes and pack it in a plastic carry bag?

Shopkeeper: – Certainly sir, I can give you potatoes, but I

  1. a) __________________________ Customer: – Strange! Then
  2. b) ____________________________________________________________?

Today  I ____________________________________________ Shopkeeper: – That’s

  1. c) ___________________________________________but I am helpless

Customer:  Why ?   Shopkeeper: –

Don’t you

  1. d) ___________________ as they increase pollution?

Customer: Thank you for _______________________________

and I Shall__________________________________________________________________

Ans.

  1. a) I cannot give you any plastic bag.
  2. b) How to carry potatoes? Should I carry then in my pockets? Today I forgot to bring my shopping bag.
  3. c) That’s your problem, sir.
  4. d) Don’t you know that the Government has banned the use of polythene .
  5. e) Reminding me about such an important point. I shall never use          plastic from now onwards.

 

 

 

PRACTICE TASK

 

Study the Dialogues and complete the dialogue meaningfully within 80 words.

 

  1. Ajay :         Could you kindly show me some batching soaps?

Salesgirl      :         Oh, ofcourse, Sir _____________________________.

Ajay             :         I would like one with the fragrance of jasmine.

Salesgirl      :         Here _____________________________________.

Ajay             :         Do you also have some perfumes?

Salesgirl      :         Which ________________________________________.

Ajay             :         Which one would you recommend?

Salesgirl      :         ____________________ most popular one.

Ajay             :         Thank you. I will take that one.

 

  1. Husband : Namrata has failed in her semester exams. What does she do all day?

Husband      :         I ___________________________________________.

Wife             :         I wonder what’s troubling her?

Husband      :         Well, _______________________________________.

Wife             :         I think we should meet her teacher.

Husband      :         I shall ________________________________________.

Wife             :         We must go together.

Husband      :         I ______________________________________________.

 

  1. Aryan : Mr. Dutta, the Racer computer which I bought from your showroom last week is a defective piece.

Suraj           :         Could you ____________________________________.

Aryan          :         There seems to be some problem with the mouse. It keeps                                                 giving wrong commands to the computer applications.

Suraj           :         ________________________________________________.

Aryan          :         I have hardly used it.

Suraj           :         ________________________________________.

Aryan          :         Yes, here it is.

Suraj           :         Please leave it behind our __________________.

Aryan          :         Thanks

 

  1. Interviewer :         ________________________________________?

Raghav        :         I have done my masters in Political Science.

Interviewer  :         ________________________________________?

Raghav        :         I did my schooling from Holy Angel Public School in                                                          Lucknow.

Interviewer  :         ________________________________________?

Raghav        :         There is limit scope for growth in that company.

Interviewer  :         ________________________________________?

Raghav        :         Thank you, Sir I shall wait for your decision.

 

  1. Customer :         I’d like to change these shoes.

Shopkeeper  :         When _______________________?

Customer     :         Someone _____________________.

Shopkeeper  :         I’m sorry, I need _________________.

Customer     :         How can I ask my friend for the receipt?

Shopkeeper  :         I am _____________________________.

 

DIARY ENTRY

 

  • This is a long answer question. The aim is to assess the students’ skills in
  • expression of ideas in clear and grammatically correct English
  • planning, organising and presenting ideas coherently
  • introducing, developing and concluding a topic
  • comparing and contrasting ideas
  • arriving at conclusions
  • presenting an argument with supporting examples
  • using an appropriate style and format
  • expanding notes into longer pieces of composition
  • creative expression of ideas

 

Guidelines

(a) Diary Entry

  • A diary entry is a first person report of an incident, behaviour of a person/persons, turn of events and the writer’s reaction to them.
  • The format, content and style of the diary entry is decided according to the occasion.
  • The format mentions the day and date.
  • The content must mention all the points related to the incident/events and the writer’s reaction to them.
  • The style may vary between personal/intimate and formal/literary.

Solved Diary Entry

Q.1

You are Vasudha. You attended an exhibition on Fashion Designing and Technology at Meena Hall, Luc­know. You were impressed by the exhibition. Write a diary entry in minimum 150 words recording your feelings and experiences.

Ans.

Lucknow

Sunday, September 21, 2014

Recently I visited the annual exhibition on Fashion Designing and Technology organised by the State Department at Meena Hall, Lucknow. I ventured into the hall by chance and not by design. However, the visitwas an eye-opener. I was amazed at the grand strides made by our country in the field of textile technology. The graphics indicated that India is one of the leading nations in exporting textiles and readymade garments. The latest spring, summer and fall fashions were on display. The technology and expertise was commendable. In the demonstration section, latest sewing machines stitched shirts in couple of minutes. There were many sophisticated machines for complicated designs and patterns. Tailoring has become an art. I felt bewildered and amused like Alice in Wonderland. I was glad that India had made a headway in the field of fashion designing and technology by storming the leading capitals abroad.

 

Q.2

 

You have recently enjoyed a Delhi Metro Ride with a close associate. The traffic Discipline and modern technology forced you to pen down your experience. Express the joyful experience through a diary entry in 50 words.

Tips : 1. Day, Date, Time

  1. Event in past tense with correct experience & expressions.

 

Ans.

12th  Sept.

Sunday : – 6.00 pm

A Joyful Ride in a Metro

 

Recently I had an opportunity to enjoy a joyful ride in Delhi Metro  with a close Friend. The discipline in the ticket Counter, the spick and span surroundings and the systematic commuters were praiseworthy. The use of technology in  announcement system, controlling the opening and closing of the doors and the speed maintenance   was stunning. My maiden experience of the ride was full of joy and excitement and I want to experience this joy ride once again.

 

 

PRACTICE TASK

 

  1. You are Anupam, a student of Tagore Public School, Jodhpur. You went on a school-conducted tour visiting a few places of historical importance. Write a diary entry in minimum 150 words giving your views on the arrangements made for this tour, the places you visited and your feeling about the tour.
  2. You witnessed a quarrel among a group of boys outside your school yesterday. Write a diary entry in minimum 150 words giving an account of the quarrel, its cause, results and your suggestions to avoid such quarrels.
  3. You have had a student from a school in Australia living with you as part of Student Exchange Programme

and he/she has just returned to her own country. Make a diary entry in a minimum of 150 words about her/his stay at your home and how you have benefited from the experience. You are_Amrita/Shobit.

  1. You are Mahi/Mehul. You are very upset that in almost all the hockey matches, held in March, 2010 in New Delhi in Major.Dhyan Chand Stadium, India lost all the matches despite being the host country. You felt very depressed as hockey is the national sport of India. Write a page in your expressing your thoughts on the declining standards of sports in India.
  2. Imagine you are a tourist guide at the Meenakshi Temple Complex. Write a diary entry describing your experi­ence of an eventful day at the complex within 150 words.

 

 

 

 

 

 

 

TYPE : 1

Test type will include gap filling to test the knowledge in the following areas :

(i) Non-finites (ii) determiners (iii) connectors (iv) modals (v) prepositions (vi) subject-verb-concord

 

INSTRUCTIONS

In the following passages, choose the most appropriate option from the ones given below to complete the passage. Write the answers in your answer sheet against the correct blank number. Do not copy the whole passage.

  1. Gravitation is the force (a) _____ holds us all down (b) _____ the surface of the earth. Anything (c) _____ upwards falls back to the earth. It is not only the earth that has the pull of gravity (d) ____ also everything else in the universe. Everything in this universe attracts (e) _____ other body to (f) _______.

 

(a)      (i) Whom                (ii) that                  (iii) whose    (iv) who

(b)      (i) into                   (ii) on                    (iii) to          (iv) in

(c)      (i) thrown              (ii) throwing           (iii) to throw          (iv) threw

(d)      (i) because             (ii) and                  (iii) so          (iv) but

(e)      (i) each                  (ii) every                (iii) either    (iv) neither

(f)       (i) themselves        (ii) herself              (iii) itself      (iv) himself

 

  1. Mark Twain was the pen name of Samuel Langhorne Clemens

(a) _____ was one of (b) ______ greatest fiction writers of America. He grew up in a small town (c) ______ the bank of the Mississippi River. (d) ______ a small boy he moved to Hannibal on the banks of the river (e) ______ he experienced (f) _______ excitement of river travel.

(a)      (i) Which               (ii) who                  (iii) that       (iv) he

(b)      (i) an                     (ii) a                      (iii) the        (iv) his

(c)      (i) on                     (ii) in                     (iii) at          (iv) upon

(d)      (i) since                 (ii) from                 (iii) to          (iv) as

(e)      (i) when                 (ii) where               (iii) whenever (iv) whence

(f)       (i) a                       (ii) his                   (iii) an         (iv) the

 

  1. Lecture as a method of teaching is as old as civilisation. It is

(a) _____ commonly practised and very widely used. In (b) ______ countries,
(c) ______ traditional and almost the only technique (d) ______ was the formal lecture. The effective use of lecture requires skilful preparation (e) ______ is incomplete (f) _______ followed by questions and answers.

 

(a)      (i) more                 (ii) much               (iii) most      (iv) many

(b)      (i) a                       (ii) the                   (iii) an         (iv) many

(c)      (i) the                    (ii) a                      (iii) every     (iv) an

(d)      (i) employ              (ii) employs            (iii) employed (iv) employing

(e)      (i) who                   (ii) which               (iii) it           (iv) whom

(f)       (i) unless               (ii) if                      (iii) fill         (iv) since

 

  1. The human body is like a machine containing (a) _____ systems (b) ______ out the processes (c) ______ life. Each system is (d) ______ up of organs (e) ______ consist of (f) _______ cells and tissues.

 

(a)      (i) little                  (ii) several             (iii) much     (iv) more

(b)      (i) to carry             (ii) carry                (iii) carries   (iv) carried

(c)      (i) for                     (ii) in                     (iii) of           (iv) with

(d)      (i) makes               (ii) making             (iii) make     (iv) made

(e)      (i) that                   (ii) who                  (iii) whose    (iv) those

(f)       (i) specialise          (ii) specialised       (iii) specialising (iv) specializes

 

  1. There is a great difference (a) _____ faith and (b) ______ people call belief. (c) ______ belief is superficial and is easily (d) ______ , faith makes us strong. We (e) ______ have faith in God (f) _______ we know that He exists.

 

(a)      (i) among               (ii) between            (iii) in          (iv) from

(b)      (i) when                 (ii) how                  (iii) what      (iv) which

(c)      (i) while                 (ii) when                (iii) since     (iv) where

(d)      (i) shaking             (ii) shaken             (iii) shake    (iv) shook

(e)      (i) can’t                  (ii) shouldn’t          (iii) can        (iv) should

(f)       (i) till                     (ii) so                     (iii) that       (iv) unless

 

  1. (a) _____ of our five sense organs (b) ______ in a perfect coordination with the other (c) ______ .

(a)      (i) every                 (ii) each                 (iii) most      (iv) much

(b)      (i) work                  (ii) worked             (iii) working (iv) works

(c)      (i) ones                  (ii) once                 (iii) one        (iv) one’s

 

  1. Jesus Christ was accused (a) _____ misleading the masses despite (b) ______ fact (c) ______ he was leading them from darkness towards. (1×3=3)

 

(a)      (i) of                      (ii) with                 (iii) from      (iv) in

(b)      (i) a                       (ii) the                   (iii) an         (iv) that

(c)      (i) this                   (ii) these                (iii) those     (iv) that

 

  1. It is (a) _____ important than anything else (b) ______ that we are a part of nature and not apart (c) ______ it.

 

(a)      (i) much                (ii) most                 (iii) many     (iv) more

(b)      (i) to remember      (ii) remember         (iii) remembering (iv) remembered (c) (i) of   (ii) from   (iii) in                    (iv) off

 

  1. Complete the passage given below choosing the correct alternatives.

The moment (a) _______ letter fell (b) _____ the mailbox, the postmaster (c) _______ to open it. It said : “God! (d) _______ the money that I asked for, only seventy pesos reached me. Send me the rest, (e) _________ I need it very much. But don’t send it to me through the mails (f) _____ the post office employees are a bunch of crooks.

 

(a)      (i) a                       (ii) his                   (iii) the        (iv) that

(b)      (i) into                   (ii) after                 (iii) in          (iv) under

(c)      (i) goes                  (ii) going                (iii) went      (iv) had gone

(d)      (i) with                  (ii) by                    (iii) of           (iv) to

(e)      (i) because             (ii) when                (iii) since     (iv) as

(f)       (i) as                     (ii) since                (iii) but        (iv) because

 

  1. Complete the passage given below choosing the correct alternatives.

A book worth (a) ______ is one which, (b) ______ read, (c) ______ auspiciousness to the reader. It (d) _______ be able to hold (e) ______ attention and be capable of (f) _______ expounded to others.

 

(a)      (i) read                  (ii) has been read   (iii) reading (iv) is being read

(b)      (i) then                  (ii) now                  (iii) when     (iv) where

(c)      (i) brought             (ii) bring                (iii) bringing          (iv) brings

(d)      (i) could                 (ii) may                  (iii) must      (iv) should

(e)      (i) its                     (ii) my                   (iii) his         (iv) their

(f)       (i) be                     (ii) becoming          (iii) being     (iv) been

 

TYPE : 2

 

Test type will include gap-filling to test the knowledge in using the words appropriately.

 

INSTRUCTIONS

 

          In the following passages, choose the most appropriate option from the ones given below to complete the passage. Write the answers in your answer sheet against the correct blank number. Do not copy the whole passage.

  1. Group (a) _________ is an (b) ________ conversation in which members of a group (c) ______ participate and exchange (d) ______ with a purpose. A problem is (e) ______ by raising questions and (f) _____ suggestions.

 

(a)      (i) discuss              (ii) discussion        (iii) to discuss        (iv) discursive (b)   (i) organise                          (ii) no organise       (iii) organised (iv) organisation (c)       (i) active           (ii) activity             (iii) actively                               (iv) act

(d)      (i) ideas                 (ii) ideal                 (iii) ideally             (iv) idiom

(e)      (i) examination      (ii) to examine        (iii) examining       (iv) examined

(f)       (i) made                 (ii) make                (iii) making            (iv) makes

 

  1. It is (a) ______ that faith cannot stand the test of (b) ______. In fact, they are not opposed to each other. (c) ______ supplement each other.

(a)      (i) belief                 (ii) believed            (iii) believing                   (iv) believe

(b)      (i) reason               (ii) reasonable        (iii) reasonably       (iv) reasoning

(c)      (i) certainty           (ii) certain             (iii) certainly                   (iv) curtained

 

  1. The mind is the (a) ______ of human life. It can be the garden of joys or the (b) ______ path to (c) ______ . One person is (d) _______ because he (e) ______ likes his occupation while another is (f) _______ because he fears death.

(a)      (i) mysterious        (ii) mystery            (iii) mysteriously    (iv) sorry

(b)      (i) secretly             (ii) secrecy             (iii) secrets            (iv) secret

(c)      (i) sorrow               (ii) sorrowful          (iii) sorrowfully      (iv) sorry

(d)      (i) depression         (ii) depress            (iii) depressed        (iv) depressive

(e)      (i) hard                  (ii) hardly              (iii) harden             (iv) harder

(f)       (i) misery               (ii) miserably         (iii)miserable         (iv) miserly

 

  1. Pollution is (a) ______ as the contamination of environment’s pure elements by the (b) ______ substances or increase in their percentage above a certain (c) ______ limit. Pollution of air is (d) _______ of pure air by noxious fumes. Pollution of water is a (e) ______ of industrial development. It is our collective (f) _______ to check pollution.

 

(a)      (i) defining             (ii) defined             (iii) define    (iv) definition

(b)      (i) harmful             (ii) harm                (iii) harmless (iv) harmfully

(c)      (i) permission        (ii) permissible       (iii) permit   (iv) permissibly

(d)      (i) contamination   (ii) contaminate     (iii) contain (iv) contained

(e)      (i) consequent        (ii) consequence     (iii) consequently (iv) consistence

(f)       (i) responsibility     (ii) responsible       (iii) response          (iv) respond

  1. Liberty is not a (a) ______ affair only but a (b) ______ contract. It is an (c) ______ of interests. In matters which do not touch others liberty, I will be (d) _______ free and won’t (e) ______ anyone’s (f) _______ .

 

(a)      (i) personally         (ii) person              (iii) personal          (iv) personality

(b)      (i) social                (ii) socially             (iii) society   (iv) socialise

(c)      (i) adjustable         (ii) adjustment       (iii) adjusted (iv) adjust

(d)      (i) completion         (ii) complete           (iii) completely (iv) completed

(e)      (i) requirement      (ii) requires           (iii) required (iv) require

(f)       (i) permissible        (ii) permission       (iii) permitted (iv) permit

 

  1. Many animals are in (a) ______ of extinction unless (b) ______ efforts are made for their (c) ______ In India, the black buck is one of such (d) _______ animals. (e) ______ killing of animals (f) _______ our ecosystem adversely.

 

(a)      (i) dangerous         (ii) endanger          (iii) danger   (iv) dangerously

(b)      (i) special              (ii) specially           (iii) speciality (iv) specialize

(c)      (i) protect              (ii) protection         (iii) protective (iv) protecting

(d)      (i) deplete              (ii) depleting          (iii) depletion (iv) depeleted

(e)      (i) indiscriminate   (ii) discriminate

(iii)indiscriminately (iv) indiscrimination

(f)       (i) affect                 (ii) effect                (iii) effective           (iv) affects

 

  1. When my maternal uncle was alive, his only son (a) ______ made plans to settle in Germany with his wife. Without (b) ______ anyone, he bought tickets and sold all (c) ______ things at home.

 

(a)      (i) quiet                 (ii) quite                (iii) quietly   (iv) quieten

(b)      (i) inform               (ii) informing         (iii) information (iv) informative

(c)      (i) moving              (ii) movable            (iii) move      (iv) moved

 

  1. The bio-diesel has (a) ______ as a (b) ______ solution to the pollution (c) ______ by emission of smoke into the atmosphere by vehicles and (d) _______ establishments. It is nothing but used vegetable oil that has been tested (e) ______. It has made a vehicle called veggie Van whichrun more than 16000 kilometres without (f) _______ the environment.

 

(a)      (i) emerge              (ii) emerged           (iii) emerging         (iv) emerges

(b)      (i) usefully             (ii) usefulness        (iii) useful              (iv) used

(c)      (i) created              (ii) creating            (iii) creation                    (iv) create

(d)      (i) industry            (ii) industrious       (iii) industrialisation(iv) industrial

(e)      (i) satisfactory       (ii) satisfy              (iii) satisfactorily   (iv) satisfying

(f)       (i) harmed             (ii) harming           (iii) harmful                    (iv) harm

 

  1. Complete the passage given below choosing the correct alternatives.

There are (a) ______ 6,000 million people (b) ______ on our planet earth. The earth provides us with the air to (c) ______ , food to eat and water to drink. It (d) _______ us materials to build home and shelter (e) ______ for clothes, and it provides us with (f) _______ other things that we need to enjoy.

 

(a)      (i) more to              (ii) more than

(iii) many more than (iv) the most of

(b)      (i) living                 (ii) are living          (iii) lives      (iv) live

(c)      (i) breathes            (ii) breathe            (iii) breathed          (iv) breathing

(d)      (i) was giving         (ii) has given          (iii) gives      (iv) has been given

(e)      (i) so well as          (ii) so much so       (iii) as well as(iv) as good as

(f)       (i) much                (ii) more                (iii) many     (iv) most

 

  1. Complete the passage given below choosing the correct alternatives.

Most (a) ______ institutions excel at (b) ______ the (c) ______ skills of students and prepare them for (d) _______ success. Teachers too (e) ______ on nurturing the (f) _______ minds in the best way possible.

 

(a)      (i) education          (ii) educating         (iii) educationally (iv) educational

(b)      (i) development      (ii) developing

(iii)developed         (iv) being developed

(c)      (i) intelligency        (ii) intelligent         (iii) intellectual (iv) intellectually

(d)      (i) academically     (ii) academy           (iii) academical (iv) academic

(e)      (i) concentration    (ii) concentrate

(iii) concentrating (iv) concentrated

(f)       (i) youth’s              (ii) young’s             (iii) young    (iv) younger

 

TYPE-3

 

          Students are providedwith a piece of conversation between two persons. The conversation is reported in a passage having four incomplete sentences. Students are required to complete the sentences in reported speech by providing their own responses. Each correct answer is worth one mark.

 

  1. Doctor : How do you feel now?

Patient : Slightly better.

Doctor : Did you take the medicines regularly?

Patient : Yes. The doctor asked the patient how

(a) _______. The patient replied (b) _______ better. When the doctor asked him (c) _______ the medicines regularly, he replied in affirmation.

 

  1. Ankit : Where are you going?

Atul : To the airport.

Ankit : May I drop you there in my car?

Atul : Thanks a lot. It will be a great help to me.

Ankit asked Atul where (a) ________. Atul replied (b) _______ airport. Ankit further asked him (c) ______ there in his car. Atul thanked him adding that it would be a great help to him.

 

  1. Tanmay : How did you spend your vacation?

Astha : I visited Kashmir with my parents.

Tanmay : Have you been there earlier too?

Astha : No, it was my first trip.

Tanmay asked Astha how (a) ______ vacation. Astha replied that (b) _______ with her parents. Tanmay further asked her if she had been there earlier too. Astha replied in negation saying that (c) ________ .

 

  1. Teacher : Why are you late?

Sumit : I missed my school bus.

Teacher : Why didn’t you leave in time?

Sumit : Sorry, I will do so from tomorrow.

The teacher asked Sumit why (a) ______ late. Sumit replied that (b) ________ school bus. When the teacher asked him why (c) _______ in time, Sumit apologetically replied that he would do so from the next day.

 

  1. Sonu : What makes you weep?

Rohan : Someone has stolen my wallet.

Sonu : Where did you keep it.

Rohan : In the back pocket of my trousers.

Sonu asked Rohan what (a) ________. He replied that someone (b) _______. Next, Sonu enquired of him where he had kept it. Rohan answered that (c) ________ in the back pocket of his trousers.

 

  1. Servant : Please give me leave for a week.

Master : Why do you need such a long leave?

Servant : I have to visit my village.

Master ; O.K. but don’t outstay your leave.

The servant requested his master (a) _______ for a week. The master asked him why (b) _______ such a long leave. At this, the servant said that he had to visit his village. The master acceded to his request but asked him (c) ______ leave.

 

 

  1. Beggar : Please give me some money.

Lady : Why don’t you do some work?

Begger : Madam, begging is also a work.

A Beggar requested a lady (a) _______. The lady asked him why (b) ______ some work. At this the beggar said (c) ______.

 

  1. John : Do you know swimming?

David : No, can you teach me how to swim?

John : Of course I can. Come to my place tomorrow at 5.00 p.m.

John asked David(a) _______. David replied in negative and asked him (b) ________ how to swim. John replied in affirmation and asked him (c) _______ the next day at 5.00 p.m.

 

  1. Principal : Why do you want a character certificate?

Student : Sir, I want to apply for a part time job.

Principal : Have you brought your progress report?

Student : No sir, I have not brought it.

The Principal asked the student (a) _________. The student replied respectfully (b) _________. The principal asked him further (c) _________. The student replied very respectfully saying that, he had not brought that.

 

  1. Madam : Have any of you seen a camel’s skin?

Boy : Yes, Madam. Madam : Where did you see it? A school mistress asked her students (a) ______ one little boy shouted out (b) _______. The madam asked him _____.

 

TYPE-4

Test type will require the students to use the correct passive forms of the verbs by choosing the appropriate options from the given ones.

 

INSTRUCTIONS

Complete the following passages by choosing the correct passive forms of the verbs given in brackets from the alternatives given below the passage. Write only your answers in the answer sheet against the correct blank number.

 

  1. Yesterday the city (a) _______ (hit) by a thunder-storm. Many trees (b) ______ (uproot) and streets at several places (c) ______ (flood) resulting in traffic jams.

(a)      (i) is hit                 (ii) was hitting       (iii) is hitting         (iv) was hit

(b)      (i) were uprooted    (ii) was uproot

(iii) were uprooting                             (iv) will be uprooted

(c)      (i) was flooding      (ii) were flooded     (iii) flooded   (iv) were flooding

 

  1. If a long journey (a) ________ (give) a good start, it is half covered. Weariness of the traveller (b) _______ (defeat) by his enthusiasm and hope. He (c) _______ (reward) with success.

 

(a)      (i) is giving            (ii) was give           (iii) is given (iv) can given

(b)      (i) is defeated         (ii) defeated           (iii) were defeated (iv) defeating (c)     (i) was rewarding                       (ii) is rewarded      (iii) is rewarding    (iv) rewarded

 

  1. Pleasure (a) ________ (enjoy) without passing through pains. Spring (b) _______ (relish) only by those whose bones (c) _______ (freeze) by unbearably cold winter.

 

(a)      (i) enjoy                 (ii) enjoys

(iii) are enjoyed      (iv) can’t be enjoyed

(b)      (i) relishing            (ii) was relished     (iii) is relished

(iv) are relishing

(c)      (i) has frozen                                      (ii) is frozen

(iii) have been frozen                           (iv) has been frozen

 

  1. Whenever I feel that I (a) ________ (leave) alone even by my shadow, my loneliness (b) _______ (dispel) by the soothing presence of someone who is everything for me. At that moment, I (c) _______ (compensate) heavily for all the losses …….. even that of my shadow.

 

(a)      (i) has left              (ii) have left

(iii) have been left (iv) have been leave

(b)      (i) has dispelled     (ii) is dispelled       (iii) was dispelled   (iv) is dispel (c)      (i) am compensated

(ii) am compensating

(iii) have compensated

(iv) was compensated

 

  1. Yesterday a U.F.O. (a) ________ (see) in the western sky over Delhi. People
    (b) _______ (horrify) to see it and started running helter skelter. It was reported to have remained suspended in the air for some time. Then it picked up speed and (c) _______ (lose) to view.

 

(a)      (i) was seeing         (ii) was saw           (iii) was seen (iv) were seen

(b)      (i) was horrify

(ii) were horrified

(iii) were horrifying

(iv) was horrified

(c)      (i) was lost             (ii) had lost            (iii) was losing (iv) has lost

 

  1. Last week a picnic trip (a) ________ (organise) by our school for class-XI. The students (b) _______ (take) to Fun and Food village. Five buses (c) _______ (hire) on the occasion.

 

(a)      (i) organised          (ii) was organised

(iii) was organising (iv) had organised

(b)      (i) had taken          (ii) was taken         (iii) were taking (iv) were taken

(c)      (i) were hired         (ii) was hired         (iii) were hiring      (iv) had hired

 

  1. Gandhiji (a) ________ (know) for practising what he preached. Our country (b) _______ (teach) the lesson of truth and non-violence by him. Thus (c) _______ (bring) into the main-stream of freedom struggle under his leadership.

(a)      (i) was knowing      (ii) had known       (iii) was known (iv) were know (b)        (i) were taught                             (ii) was taught       (iii) is teaching      (iv) had taught

(c)      (i) was brought      (ii) was being brought

(iii) had brought     (iv) was bringing

 

  1. Even the deepest wounds, that (a) ________ (give) over a long time by all the cruelties of the world (b) _______ (heal) by a few words of sympathy and love that (c) _______ (speak) by someone who is not apart from you but a part of you.

 

(a)      (i) has been given   (ii) have been giving

(iii) has been give   (iv) was giving

(b)      (i) is healed           (ii) are healed        (iii) will heal          (iv) was healing

(c)      (i) are spoken         (ii) is speaking       (iii) are speaking (iv) have spoken

 

TYPE : 5

 

Test type will require the students to attempt questions based on one of the following areas of learning.

(i)       Filling in the blanks with suitable clauses to complete a dialogue

(ii)      Re-ordering the jumbled up words and phrases to make meaningful sentences.

 

Complete the dialogue by filling up the blanks with the most appropriate options from those given.

 

  1. Abhay : Give me a book (a) _________

Kamal : I cannot give you any book today (b) _________

Abhay : Please tell me (c) _________

 

(a)      (i) It is very interesting

(ii) that is very interesting

(iii) Which is very interesting?

(iv) Who is very interesting?

 

(b)      (i) which is interesting.

(ii) that was interesting.

(iii) that had been interest

(iv) who is very interesting

 

(c)      (i) which can come

(ii) when to come.

(iii) when is coming

(iv) when it is coming

 

  1. Police man: Where (a) _________

Rahul : I am coming from my office.

Policeman : Where (b) _________,

Rahul : I work in Nokia Care Centre No. 10.

Policeman : But, why (c) _________ late?

Rahul : Today there was a party in the office.

 

(a)      (i) from are you coming?

(ii) are you coming from?

(iii) where have you been coming from?

(iv) had you been coming from?

 

(b)      (i) you work

(ii) did you work

(iii) do you work

(iv) have you been working

 

(c)      (i) you should so

(ii) you may so

(iii) you will be so

(iv) are you so.

 

  1. Shweta : Will you tell me (a) _________?

Shubham : I wanted to tell you (b) _________

Shweta : In fact, I always wanted to drive a car (c) _________

 

(a)      (i) Why have you sold that car

(ii) why you sold that car

(iii) where you are selling this car

(iv) why did you sell that car

 

(b)      (i) that it was very slow

(ii) if it is very slow

(iii) if it was very slow

(iv) that they were very slow

 

(c)      (i) which run faster than the winds

(ii) which runs faster than the winds

(iii) that it runs faster than the winds

(iv) which is running faster than the winds

 

  1. Vani : Congrats!

Varun you have done very well in English.

Varun : Thank you very much.

Vani : Tell me (a) _________.  (b) _________.

Varun : Certainly yes, I studied from ABC English Centre.

Vani : I would like to know (c) _________

 

(a)      (i) why have you managed to score 98%

(ii) how you have managed to score 987.

(iii) how have you managed 98%

(iv) how come did you manage to score 98%

 

(b)      (i) Did you not take any class?

(ii) Where you took coaching?

(iii) Where are how did you take coaching?

(iv) Who guide you in coaching?

 

(c)      (i) what is your future plans

(ii) what will you plan in future

(iii) what are your future plans

(iv) how can you plan for your future

 

  1. Sonu : Have you ever visited Assam?

Monu : Yes, (a) _________

Sonu : Do you know (b) _________?

Monu : It is a beautiful place and it is very famous for tea ……

Sonu : (c) _________?

 

(a)      (i) when I was very youngest

(ii) when I was very young

(iii) when I became very young

(iv) when I was youth

(b)      (i) What was it famous for?

(ii) Why was it famous for

(iii) what is it famous for

(iv) when is it famous for

(c)      (i) how you reached there

(ii) how could one reach there

(iii) how did you reach there

(iv) do you know how to reach there.

  1. Niru : Anita, please tell me (a) _________.

Anita : I think (b) _________.

Niru : OK, it will suit me but make sure (c) _________.

 

(a)      (i) how the house warming ceremony takes place?

(ii) what the house warming ceremony take place?

(iii) when the house warming ceremony will take place?

(iv) when will the house ceremony take place.

 

(b)      (i) how it will take place in the morning?

(ii) why it will take place in the morning?

(iii) that it will take place in the morning.

(iv) that it can take place in the morning.

 

(c)      (i) that I can accompany you.

(ii) that I accompany you.

(iii) how I can accompany you

(iv) that I am accompany you.

 

  1. Mother : Pankaj, why are you so late from school today?

Pankaj : I visited my friend today.

Mother : Your friend Deepak just called

Pankaj : Did you (a) _________

Mother : She wanted to know (b) _________

Pankaj : I had told her that (c) _________

 

(a)      (i) took any message from her?

(ii) take any message from her?

(iii) any message from her?

(iv) taken any message from her?

 

(b)      (i) Whether you will be going to Radha’s birthday party.

(ii) Whether will you be going to Radha’s birthday party.

(iii) Whether shall you be going to Radha’s birthday party.

(iv) Whether may you be going to Radha’s birthday party.

 

 

(c)      (i) I will confirm my programme on telephone.

(ii) I shall confirm my programme on telephone.

(iii) I would confirm my programme on telephone.

(iv) I must confirm my programme on telephone.

 

  1. Neha : Good morning, Tanu! (a) _________?

Tanu : Good morning Neha. I am coming from medical store.

Neha : I hope all is well. (b) _________ so early?

Tanu : I went there to bring some medicines for mother.

Neha : (c) _________?

I hope it is not severe.

Tanu : She had a minor heart attack but now she is O.K.

 

(a)      (i) Why are you up to day

(ii) here you went away

(iii) where are you coming from

(iv) where do you come from

 

(b)      (i) how you went there

(ii) why you went there

(iii) why did you go there

(iv) how have you gone there

 

(c)      (i) What has happened to her

(ii) which has happened to her

(iii) why had it happened to her

(iv) how it happened to her.

 

 

Type : 6

 

Re-ordering of words/phrases into meaningful sentences

 

The words and phrases in the following sentences are jumbled up. Rearrange the words to make meaningful sentences.

 

  1. (a) place / our lives / music / important / has / in / an

(b) body and / activates / our / it / mind

(c) a / is / art / also / regarded / as / fine / it

 

  1. (a) its / urbanisation / in India / everywhere / has / tentacles / spread

(b) only / the country / land / of / remains / villages / in name / in

(c) policy-makers / numerous / this / posed / challenges / has / the / for

 

  1. (a) religious / of / or spiritual / form / a / meditation is / contemplation

(b) basis / religions / most of the / of / the / is / it

(c) one’s own mind / processes / detached observation / and / is /

its / it / of

 

  1. (a) hardly / koalas are / ever drink / animals that / water

(b) water supply / they get / they eat / from / all their / the leaves

(c) can eat / everyday / each / one and a half kilograms / adult koala

/ upto / of leaves

 

  1. (a) are / eating / small / birds / insects / wagtails

(b) eight / of / wagtails / there / about / are / species

(c) to / are / related / they / closely / pipits

 

  1. (a) the balcony / and / a beam / fell / of moonlight / on the bed / stepped over

(b) situation / on the floor, / the / considering / sat / up / I

(c) I / I / the money, / if / took / 10:30 express / catch / could / the /

Lucknow / to

 

  1. (a) down / the / again / dog / looked / at / I / little

(b) to / never / food / he / been / refuse / had / known

(c) right / at any / or / would / a / tackle / meal / hour / day / the / he / of

  1. (a) about / worth of / were / fifteen thousand / in the / jewels /

pounds / there / safe

(b) one by / if / one, / sold / expected / get / he / to / at least / five

thousand / he / them

(c) three / were / interesting / there / books / very / for sale / in the /

coming up / autumn

 

  1. (a) his / him / him / to / tried / and / eject / landlord / disliked

(b) Griffin / to / the / set / in / fire / revenge / house

(c) clothes / to / away / his / get / remove / without / had to / seen / he

/ being

 

  1. (a) ear / of / to / all / her / a sudden / a / Mrs. Hall / sniff / heard / close

(b) on the / the hat / a moment / bedpost / later / her / itself / into /

and / leapt up / dashed / face

(c) chair / then / alive / bedroom / the / became

 

Type : 7 Completing Headlines

 

Complete the headlines by choosing correct answers from the options given below.

 

  1. 300 DIED IN INDONESIA.

More than 300 people _____________ in a quake-triggered Tsunami in Mentamai Islands in Indonesia.

(i) are died

(ii) have been reported dead

(iii) have reported dead

(iv) report to be dead

 

  1. TODAY 1600 BLUELINE BUSES TO GO OFF ROAD.

The _____________ a part of history with the notification to do away with private bus service.

(i) 1600 odd blueline buses are become

(ii) 1600 odd blueline buses had become

(iii) 1600 odd blueline buses have become

(iv) 1600 odd blueline buses will become

 

  1. MAN ARRESTED FOR SELLING DRUGS IN A COLLEGE CAMPUS.

A 40 year man _____________ in South Delhi college.

(i) will be arrested for selling drugs

(ii) is arrested for selling drugs

(iii) was arrested for selliing drugs

(iv) has arrested for selliing drugs.

 

  1. NOIDA ON DECONGESTION DRIVE.

Three new elevated roads and four new underpasses __________ .

(i) are constructed to decongest Noida

(ii) have constructed to decongest Noida

(iii) will be constructed to decongest Noida

(iv) has been constructed to decongest Noida

 

  1. NOW KIDNEY DAMAGE CAN BE DETECTED EARLY.

A new technique for _____________ many lives in future.

(i) early detection of Kidney Diseases could save

(ii) early detection of Kidney Diseases should save

(iii) early detection of Kidney Diseases has saved

(iv) early detection of Kidney Diseases have saved

 

  1. LOK ADALAT SETTLES 406 CASES

As many as 406 cases _____________ in the Rural Lok Adalats held in various districts of Haryana on this Sunday.

(i) have been settled

(ii) were settled

(iii) had been settled

(iv) had settled

 

  1. CHINA DEVELOPS MEDICAL ROBOT

A Polytechnic University in China _____________ a medical robot.

(i) developed

(ii) has been developed

(iii) has developed

(iv) will develop

 

 

  1. 7 CRORE SEIZED IN POLL-BOUND ANDHRA

Election Commission Officials _____________ Rs. 7 crore in unannounced cash from various parts in Andhra Pradesh.

(i) have been seized

(ii) were seized

(iii) have seized

(iv) had seized

 

  1. U.K. TO OPEN 2 MORE MISSIONS IN INDIA London :

To boost trade and investment the U.K. on Wednesday _____________ the opening of two more missions in India.

(i) has announced

(ii) will announced

(iii) is announcing

(iv) announced

 

  1. DU TO CHECK FAKE DOCUMENTS : After being hit by a fake marksheet scam, the Delhi University _____________ special instructions to colleges to make their verification process foolproof.

(i) is issuing

(ii) has issued

(iii) can issue

(iv) issued

 

  1. BIRD HONOURED London : English umpiring Great Herald ‘Dickie’ Bird _________ with an honour at Buckingham Palace in recognition of his outstanding sporting.

(i) has been presented

(ii) is presented

(iii) has been given

(iv) was presented

 

  1. YOUNGMAN KILLED IN MISHAP New Delhi :

A young man _____________ and his brother sustained serious injuries in an accident involving police vehicle on Friday.

(i) has been killed            (ii) had been killed

(iii) got killed                   (iv) was killed

Type – 8 Editing (Error Finding)

The following passage has not been edited. There is an error in each line against which a blank is given. Write the incorrect word and the correction in your answer sheet against the correct blank number as given in the example. Remember to underline the word that you have supplied.

 

  1. Ten of thousand of bat emerge e.g. emerge – emerged

Again and again he would peep                     (a) ………………

a amazing sight. I learnt a                            (b) ………………

few fact about these creatures. The baby       (c) ………………

bats is known as pups. Each                        (d) ………………

mother bat delivers only one pups                 (e) ………………

They are usually born on June/July             (f) ………………

 

  1. An one-eyed man was travelling e.g. An – A

through a bus one day. He was                     (a) ………………

carrying a huge bag on him                          (b) ………………

shoulder. Anyone sitting next to him             (c) ………………

said, “Why didn’t you keep your bag              (d) ………………

beneath the seat. The man smiled                (e) ………………

and said, “It is too big to be kept their.”         (f) ………………

 

  1. In the process to learning e.g. to – of

self-help is the better help                            (a) ………………

Depend always on others,                             (b) ………………

even on teachers or parents                          (c) ………………

is an sign of weakness.                                 (d) ………………

In the Mahabharta their is a                        (e) ………………

wonder character of Eklavya which               (f) ………………

is an ideal for modern students.

 

  1. Thanks to the current interest for fitness e.g. for – in

and health that people are taking                 (a) ………………

In each locality we find this days                  (b) ………………

some centres and clubs is springing             (c) ………………

up and they are do well in their                    (d) ………………

new venture. Because unfortunately they      (e) ………………

are not within easy reach for all.                  (f) ………………

  1. She lost her father when she is still e.g. is – was

a child. Her uncle looked at all the property (a) ………………

that she inherit from her father.                   (b) ………………

Since a few years her uncle worked very       (c)………………

sincerely. Afterwards he thinks, “This           (d) ………………

property should make me rich. How long       (e) ………………

should I kept serving my niece? I                  (f)………………

must do something.

 

  1. Automated Teller Machines (ATM) has e.g. has – have

revolutionised banking and made life easiest.       (a) ………………

Bank customers could now withdraw            (b) ………………

money of their account anytime and             (c) ………………

somewhere in their own country                   (d) ………………

or even from the world. However                   (e) ………………

like any other electrical gadget                     (f) ………………

they can also malfunction.

 

  1. “So you want other day e.g. other – another

of”, shouted the office manager                     (a) ………………

on his clerk. I am anxious                            (b) ………………

to heard what excuse you have                     (c) ………………

now. You are gone for your                           (d) ………………

grandfather funeral twice                             (e) ………………

already.” The clerk smile sheepishly             (f) ………………

and said, “Today my grandmother is

getting married again.

 

  1. The concern in global warming e.g. in – over

is not that its happening but                        (a) ………………

that it’s been hastened by modern                (b) ………………

man. It is also occuring in a far                    (c) ………………

greater rate then the natural                        (d)………………

evolutionary process. We are cut                   (e) ………………

down trees, forcing an extinction of               (f) ………………

many species and polluting the

environment.

Incorrect        Correct

  1. Leena was in her annual visit e.g.        in             on

to her uncle house. She always                     (a) …………………….

enjoys it because she was allowed                (b) ………….. ………….

to spend much fo the day                              (c) ………….. ………….

down under the mango grove. Leena’s           (d) ………….. ………….

uncle was a friendship of the man                (e) ………….. ………….

who owned the grove and he always gave a   (f) ………….. ………….

especial rate.                                               Incorrect               Correct

 

  1. This year Reema’s aunt joined him, e.g.       him          her

but together they set up across the fields to the (a) ………….. ………….

grove. The branches of the trees is covered    (b) ………….. ………….

with fruits, and so bowing down                   (c) ………….. ………….

with the weightage that they almost             (d) ………….. ………….

touched the ground. They spend hours picking (e) ………….. ………….

fruit, eating many of it and sleeping in the    (f) ………….. ………….

shade.                                                         Incorrect               Correct

 

  1. There lived a wisely old man in Purkul, e.g.        wisely      wise Dehradun. The villagers looked up in him and                                    (a) ………….. ………….

approached him for all there problems. Three             (b) ………….. ………….

naughty boy : Amar, Naveen and Praveen want to (c) ………….. ………….

test the oldman’s wisdom, one finest morning (d) ………….. ………….

they caught a butterfly when playing            (e) ………….. ………….

in the garden. Amar has the butterfly in his (f) ………….. ………….

hand.

Incorrect Correct

  1. Our school is conducted a cultural e.g.        conducted conducting fiesta on the 26th in this month.                                      (a)         ………….. ………….

The events for

competition includes music, dance               (b) ………….. ………….

and skit. Each school are requested to send (c) ………….. ………….

not more than fifteen participants. We

must appreciate if the students of                 (d) ………….. ………….

his prestigious institutions participate          (e)

in a competition and make                           (f) ………….. ………….

our a grand success.

 

Type : 9 Editing (Omissions)

          In the following passage one word has been omitted in each line. Write the missing word along with the word that comes before and the word that comes after it in your answer sheet against the correct blank number. Ensure that the word that forms your answer is underlined. The first one has been done for you.

 

  1. A man getting very impatient. e.g. man was getting

Again and again he would peep the                            (a) ………………

doctor’s room to see many patients                             (b) ………………

were left. After a long time his turn                            (c) ………………

came, he got up said, “Sorry doctor                            (d) ………………

I don’t have problem now. This long                            (e) ………………

wait has cured headache. I’m here                             (f) ………………

only to thank you.

 

  1. Fresh vegetables be cooked to perfection e.g. vegetables can be

in the microwave.They retain their                             (a)………………

full flavour, nutrients colour. Vegetables                    (b) ………………

can be cooked in casserole dishes lids                        (c) ………………

stirring once the cooking period.                                (d) ………………

Roasting boiling bags can also be used                       (e) ………………

for some vegetables with ends                                    (f) ………………

loosely closed allowing a bit of steam to escape.

 

  1. Now a days most the children e,g. most of the

get an opportunity go on school                                  (a) ………………

trips. They jump the chance                                      (b) ………………

to be away home for a                                                (c) ………………

week two. Some children, specially                            (d) ………………

those leaving home for first time                                (e) ………………

very much miss parents.                                           (f) ………………

 

  1. India a land of great diversity. e.g. India is a

There is diversity only in its                                       (a) ………………

geographical features also in the                                (b) ………………

race that its people belong, in                                    (c) ………………

the religion that profess and                                      (d) ………………

the languages that they. There                                   (e) ………………

is, however, a unity in its diversity                            (f) ………………

and this help to unite its people into one nation.

 

  1. All living things to breathe e.g. things have to

to survive. Each species has                                      (a) ………………

its peculiar way. Some                                               (b) ………………

breath lungs while others                                          (c) ………………

with gills or through skin.                                         (d) ………………

The respiratory system man is highly                         (e) ………………

developed and consists mainly the                             (f) ………………

respiratory passage and the lungs.

 

  1. Each and everyone wishes to healthy e.g. to be healthy

but very few ready to make efforts                              (a) ………………

in this direction. Most the people                               (b) ………………

give excuse that they busy and                                  (c) ………………

can’t spare any minute exercise.                                (d) ………………

Others believe in eating too. They                              (e) ………………

think that they get healthy by eating                          (f) ………………

all the time.

 

  1. Now in same village there lived e.g. in the same

a wicked old man his wife,                                         (a) ………………

not bit sensitive and kind, had                                   (b) ………………

always kicked and scolded dogs                                 (c) ………………

whenever anybody passed house.                               (d) ………………

Hearing their neighbour’s good luck                           (e) ………………

they coaxed the dog into their garden                         (f) ………………

and set before bits of fish hoping

he would find treasure for them.

 

  1. The shop had certain quiet distinction. e.g. had a certain

There was no sign upon other than                            (a) ………………

the name Gessler Brother; and                                  (b) ………………

in the window pairs of boots.                                     (c) ………………

He made only what ordered,                                       (d) ………………

and what made never failed to fit.                              (e) ………………

Such boots he made                                                  (f) ………………

seemed to me mysterious and wonderful.

 

  1. Hilary Clinton is 67th U.S. Secretary of State. e.g. is the 67th

She embanked a career in law graduating from          (a) ………………

Yale Law School. She elected as Senator for               (b) ………………

Nework state 2000 Obama’s Secretary and as                            (c) ………………

of state. She is the former first lady                           (d) ………………

to serve in President’s Cabinet. As the                        (e) ………………

She is responsible carrying out the                            (f) ………………

President’s foreign policies.

 

  1. Necessity, they say, is mother of invention e.g. is the mother

and Indian doctors have quite creative when                              (a) ………………

hamstrung by few no tools to perform specific                            (b) ………………

surgeries. They simply design themselves                   (c) ………………

at one fourth price they are sold abroad.                    (d) ………………

In fact, some their innovations are                             (e) ………………

priced at as much hundreds of dollars abroad.           (f) ………………

 

  1. A customer walked a bakery e,g, walked into a

and complained the bread he                                     (a) ………………

had bought previous day had                                     (b) ………………

too baking powder in it.                                             (c) ………………

The man at the counter told that                               (d) ………………

that because they only served                                    (e) ………………

those people who to rise and shine.                            (f) ………………

 

  1. The fact that dance an art form is a e.g. dance is an

well-known fact. Although dance a                             (a) ………………

therapy is not known many. Dance therapy                (b) ………………

involves a synthesis of the grace and vigour               (c) ………………

Indian classical and folk dance movement into           (d) ………………

innovative and holistic therapy. It brings the inner     (e) ………………

feelings in the participants and can help them                           (f) ………………

develop a healthy personality.

  1. Avvaiyar was ninth century Tamil e.g. was a ninth

poet greatly revered for wisdom. She                          (a) ………………

travelled foot from place to place, spreading               (b) ………………

noble thoughts. One day, hungry tired with                (c) ………………

her travels sat under a tree to rest.                            (d) ………………

All of sudden, she heard a voice calling                      (e) ………………

out to her, “Patti, you want some fruits?”                   (f) ………………

 

  1. Meditation is a system, nor it is e.g. is not a

a part of organised religion. It                                    (a) ………………

does not even require God a reference point.                              (b) ………………

Because, the state self immersion, called                   (c) ………………

Bakhadi by the suffis, no object to focus on. Meditation (d) ………………

is pure metaphysics. Jani said, “Bekhadi is                (e) ………………

intoxication of the soul the inebriation of the soul.     (f) ………………

 

  1. There are several reasons a headache. e.g. reasons for a

Physical, emotional, mental factors, anxiety               (a) ………………

and tension are few. Sometimes                                 (b) ………………

headache can a signal of an underlying                      (c) ………………

disease. More medicines, yoga therapy                       (d) ………………

eminently suits any need. Yoga a                               (e) ………………

comprehensive mode of culturing the body the mind. (f) ………………

 

  1. The aim of Gita is not to teach e.g. of the Gita

a theory as to enforce the practice dharma.                (a) ………………

Dharma is what promotes worldly prosperity,                            (b) ………………

spiritual freedom. Kurushetra also called Tapakshetra,              (c) ………………

the field of penance discipline. War is                        (d) ………………

a punishment and cleaning mankind. God is a           (e) ………………

judge as well the redeemer. He destroys and                              (f) ………………

creates.

 

Solution

Section : C

 

Type : 1

  1. (a) (ii) that

(b) (iii) to

(c) (i) thrown

(d) (iv) but

(e) (ii) every

(f) (iii) itself

  1. (a) (ii) who

(b) (iii) the

(c) (i) on

(d) (iv) as

(e) (ii) where

(f) (iv) the

  1. (a) (iii) most

(b) (iv) many

(c) (i) the

(d) (iii) employed

(e) (ii) which

(f) (i) unless

  1. (a) (ii) several

(b) (i) to carry

(c) (i) for

(d) (iv) made

(e) (i) that

(f) (ii) specialised

  1. (a) (ii) between

(b) (iv) what

(c) (i) while

(d) (ii) shaken

(e) (iv) should

(f) (iii) that

 

  1. (a) (ii) each

(b) (i) work

(c) (i) ones

  1. (a) (i) of

(b) (ii) the

(c) (iv) that

  1. (a) (iv) more

(b) (i) to remember

(c) (ii) from

  1. (a) (iii) the

(b) (i) into

(c) (iii) went

(d) (iii) of

(e) (i) because

(f) (i) as

  1. (a) (ii) reading

(b) (iii) when

(c) (iv) bring

(d) (iv) should

(e) (iii) his

(f) (iii) being

 

Type : 2

 

  1. (a) (ii) discussion

(b) (iii) organised

(c) (iii) actively

(d) (i) ideas

(e) (iv) examined

(f) (iii) making

  1. (a) (ii) believed

(b) (i) reason

(c) (iii) certainly

  1. (a) (ii) mystery

(b) (iv) secret

(c) (i) sorrow

(d) (iii) depressed

(e) (ii) hardly

(f) (iii) miserable

  1. (a) (ii) defined

(b) (i) harmful

(c) (i) permissible

(d) (ii) contamination

(e) (ii) consequence

(f) (i) responsibility

  1. (a) (iii) person

(b) (i) social

(c) (ii) adjustment

(d) (iii) completely

(e) (iv) require

(f) (ii) permission

  1. (a) (ii) endanger

(b) (i) special

(c) (ii) protection

(d) (ii) depleting

(e) (i) indiscriminate

(f) (iii) quietly

  1. (a) (ii) informing

(b) (i) movable

  1. (a) (ii) emerged

(b) (iii) useful

(c) (i) created

(d) (iv) industrial

(e) (i) satisfactory

(f) (iv) harming

  1. (a) (ii) more than

(b) (i) living

(c) (ii) breathe

(d) (iii) gives

(e) (iii) as well as

(f) (iii) many

  1. (a) (i) education

(b) (ii) developing

(c) (iii) intellectual

(d) (iv) academic

(e) (ii) concentrate

(f) (iii) young

 

Type : 3

 

  1. (a) (ii) he felt then

(b) (iv) he felt slightly

(c) (iii) he had taken

  1. (a) (iii) he was going

(b) (iv) that he was going to the

(c) (i) if he might drop him

  1. (a) (iii) she had visited

(b) (iv) she had visited

(c) (ii) it had been her first trip

  1. (a) why he was late.

(b) he had missed his bus

(c) why he had not left.

  1. (a) made him weep.

(b) had stolen his wallet.

(c) breathe

  1. (a) to give him leave for a week.

(b) he needed

(c) not to outstay

  1. (a) to give him some money

(b) he didn’t do any work

(c) that beginning was/is also a work.

  1. (a) it he knew swimming.

(b) if he could teach him.

(c) to come to his place.

 

  1. (a) why he wanted a character certificate.

(b)  that he wanted to apply for a part time job.

(c) Whether he had brought his progress report.

  1. (a) Whether anyone of them had seen a camel’s skin

(b) that anyone of them had seen a camel’s skin.

(c) where had he seen it.

 

Type : 4

 

  1. (a) (iv) was hit

(b) (i) were uprooted

(c) (ii) were flooded

  1. (a) (iii) is given

(b) (i) is defeated

(c) (ii) is rewarded

  1. (a) (iv) can’t be enjoyed

(b) (iii) is relished

(c) (iii) have been frozen

  1. (a) (iii) have been left

(b) (ii) is dispelled.

(c) (i) was compensated.

  1. (a) (iii) was seen

(b) (ii) were horrified.

(c) (i) was lost.

  1. (a) (ii) was organised.

(b) (iv) were taken.

(c) (i) were hired.

  1. (a) (iii) was known.

(b) (ii) was taught.

(c) (i) was brought.

  1. (a) (i) has been given.

(b) (i) is healed.

(c) (i) are spoken

 

 

Type : 5 Dialogue Completion

 

  1. (a) (iii) which is very interesting.

(b) (i) which is interesting

(c) (iii) when to come.

  1. (a) (ii) are you coming from.

(b) (iii) do you so.

(c) (iv) are you so.

  1. (a) (iv) why did you sell the car.

(b) (i) that it was very slow.

(c) (i) which run faster than the winds.

  1. (a) (iii) how have not managed 98%

(b) (i) did you not take any class.

(c) (iii) what are your future plans.

  1. (a) (ii) when I was very young.

(b) (iii) what is it famous for.

(c) (iv) do you know how to reach there.

  1. (a) (iii) when the house warming ceremony will take place.

(b) (iii) that it will take place in the morning.

(c) (ii) that I accompany you.

  1. (a) (ii) take any message from her ?

(b) (i) whether you will be going to Radha‘’s birthday party.

(c) (iii) I would confirm my program on telephone.

  1. (a) (iii) where are you coming from.

(b) (iii) why did you go there.

(c) (i) what has happened to her.

 

Type : 6

 

  1. (a) Music has an important place in our lives

(b) It activates our body and mind

(c) It is also regarded as a fine art

  1. (a) Urbanization has spread its tentacles every-where in India.

(b) In name of country only land remain in the villages.

(c) This has posed numerous challenges for policy makers.

  1. (a) Meditation is a form of religion or spiritual contemplation.

(b) It is the basis of most of the religions.

(c) It is detached observation processes of one’s own mind.

  1. (a) Koalas are animal that hardly drink water.

(b) They get all their water supply from the leaves they eat.

(c) Each adult Koalas can eat upto one and a half kilograms of leaves everyday.

  1. a. Small wagtails birds are eating insects.
  2. There are about eight species of wagtails.
  3. They are closely related to pipits.
  4. (a) A beam of moonlight fell on the bed and stepped over the balcony.

(b) I sat up on the floor, considering the situation.

(c) If I took the money I could catch the 10:30 express to Lucknow.

  1. (a) I looked down at the little dog again.

(b) He never had known to refuse food.

(c) If any hour of the day he would be right to take a meal.

  1. (a) There were jewels about fifteen thousand worth of pounds in safe.

(b) He expected to get at least five thousand if he add them one by one.

(c) There were three interesting books for sale coming up in the autumn.

  1. (a) His landlord disliked him and tried to eject him.

(b) Griffin set fire to the house in revenge

(c) He had to remove his cloths to get away without being seen.

  1. (a) All of sudden Mrs. Hall heard a sniff close to her ear.

(b) A moment later the hat on the bedpost leapt up and dashed itself into her face.

(c) Then the bedroom chair become alive.

 

Type : 7 Completing Headlines

  1. (ii) have been reported dead
  2. (iv) 1600 odd blue line buses will become
  3. (iii) was arrested for selling drugs
  4. (iii) will be constructed to decongest Noida.
  5. (i) early detection of kidney diseases could save.

6        (i) have been settled.

  1. (iii) has developed.
  2. (iii) have seized.
  3. (iv) announced.

10      (ii) has issued.

  1. (i) has been presented.
  2. (iv) was killed.

 

Type : 8 Editing (Error Finding)

 

  1. (a) were – was

(b) a – an

(c) fact – facts

(d) is – are

(e) pups – pup

(f) on – in

  1. (a) through – by

(b) him – his

(c) Anyone – Someone

(d) didn’t – don’t

(e) beneath – under

(f) their – there

  1. (a) better – best

(b) Depend – Depending

(c) or – and

(d) an – a

(e) their – there

(f) which – who

  1. (a) that – which

(b) this – these

(c) is – are

(d) do – doing

(e) because – but

(f) for – of

  1. (a) at – after

(b) inherit – inherited

(c) since – for

(d) thinks – thought

(e) should – can

(f) kept – keep

 

  1. (a) easiest – easier

(b) could – can

(c) of – from

(d) some where – any where

(e) from – in

(f) gadget – gadgets

  1. (a) of – off

(b) on – at

(c) heard – hear

(d) are – have

(e) grand father – grand father’s

(f) smile – smiled

  1. (a) its – it’s

(b) been – being

(c) in – at

(d) then – than

(e) cut – cutting

(f) an – the

  1. (a) uncle – uncle’s

(b) enjoys – enjoyed

(c) much – most

(d) under – in

(e) friendship – friend

(f) and – so

  1. (a) but – and

(b) is – are

(c) bowing – bowed

(d) weightage – weight

(e) fruit – fruits

(f) it – them

  1. (a) in – at

(b) there – their

(c) want – wanted

(d) finest – fine

(e) when – while

(f) has – had

  1. (a) in – of

(b) includes – include

(c) are – is

(d) must – will/shall

(e) his – your

(f) our – it

 

Type : 9 Editing (Omissions)

 

  1. (a) peep into the

(b) see how many

(c) time when his

(d) up and said

(e) have any problem

(f) cured my headache

  1. (a) they can retain

(b) nutrients and colour

(c) dishes with lids

(d) once during the

(e) Roasting or boiling

(f) with the ends

  1. (a) opportunity to go

(b) jump at the

(c) away from home

(d) week or two

(e) for the first

(f) miss their parents

  1. (a) is not diversity

(b) features but also

(c) belong to in

(d) they speak.There

(e) however still a

(f) this helps to

 

  1. (a) survive and each

(b) its own peculiar

(c) breath from lungs

(d) through their skin

(e) system in man

(f) mainly of the

  1. (a) few are ready

(b) most of the

(c) they are busy

(d) minute for exercise

(e) too much.

(f) they can get

  1. (a) man and his

(b) not a bit

(c) scolded the dogs

(d) passed his house

(e) hearing from their

(f) before him bits

  1. (a) upon it other

(b) name of Gessler

(c) window a few pairs

(d) what was ordered

(e) what he made

(f) boots which made

  1. (a) embanked for a

(b) she was elected

(c) state in 2000

(d) of the state

(e) in the president’s

(f) responsible for carrying

  1. (a) have a quite

(b) by a few

(c) design for themselves

(d) fourth of price

(e) some of their

(f) much as hundreds

 

  1. (a) complained that the

(b) brought the previous

(c) too much baking

(d) told him that

(e) that was because

(f) who have/want to

  1. (a) dance is a

(b) known to many

(c) vigour of Indian

(d) into an innovative

(e) and a holistic

(f) them to

  1. (a) for her wisdom

(b) traveled on foot

(c) hungry and tired

(d) travels she sat

(e) of a sudden

(f) Patti, do you

  1. (a) of an organised

(b) God for a

(c) state of self

(d) suffix is no

(e) is an intoxication

(f) soul and the

  1. (a) emotional and mental

(b) are a few

(c) can be a

(d) medicines and/but yoga

(e) yoga is a

(f) body and / with the mind

  1. (a) practice of dharma

(b) promotes the worldly

(c) kurukshetra _____ also

(d) penance and discipline

(e) cleaning of mankind

(f) well as the